Business Law and Practice MCQS Flashcards

1
Q

Which ONE of the following is correct?

A) A partnership can be described as two or more people forming an intention to work together with a view to profit.

B) LLPs are required to file annual accounts at Companies House.

C) A limited partnership can be formed without any required formalities.

D) A sole trader requires a certificate of incorporation to start trading.

A

B) LLPs are required to file annual accounts at Companies House.

How well did you know this?
1
Not at all
2
3
4
5
Perfectly
2
Q

Which ONE of the following correctly represents the characteristics of an LLP?

A) The LLP is a separate legal entity and enters into contracts on its own behalf. All partners have limited liability. Only professional firms such as law firms and accountancy firms may become LLPs.

B) The LLP is a separate legal entity and enters into contracts on its own behalf. All partners have limited liability. Although LLPs must be registered at Companies House there are no further filing requirements for LLPs.

C) The LLP is a separate legal entity and enters into contracts on its own behalf. Certain partners can have limited liability, but these partners must not be involved in the management of the business and are known as sleeping partners.

D) The LLP is a separate legal entity and enters into contracts on its own behalf. All partners have limited liability. LLPs must be registered at Companies House and must submit annual accounts.

E) The LLP is a separate legal entity and enters into contracts on its own behalf. All partners have limited liability. An LLP is a better structure for investment than a traditional partnership as it can issue shares and therefore attract investment.

A

D) The LLP is a separate legal entity and enters into contracts on its own behalf. All partners have limited liability. LLPs must be registered at Companies House and must submit annual accounts.

How well did you know this?
1
Not at all
2
3
4
5
Perfectly
3
Q

Which ONE of the following is correct in relation to a traditional partnership?

A) A partnership is formed when two or more persons sign a Partnership Agreement.

B) In the absence of an express written Partnership Agreement, the Partnership Act 1890 provides that all profits are shared equally, partners are jointly and severally liable for debts of the partnership and all partners can participate in management.

C) In the absence of express agreement, the Partnership Act 1890 provides that all profits are shared equally, partners are jointly and severally liable for debts of the partnership and all partners can participate in management.

D) Under the Partnership Act 1890, in the absence of express agreement, partners will cease to be liable for debts incurred by the partnership at a time when they were partners once they leave the partnership.

E) Partnerships must be registered at Companies House but there is no requirement to file annual accounts.
Correct
Correct. Although any express agreement will override the default provisions of the Partnership Act 1890 (written or oral), partners should ensure that they draw up a written express Partnership Agreement to regulate their partnership as they require which will provide for certainty. The default provisions of the Partnership Act 1890 are rarely appropriate for modern businesses.

A

C) In the absence of express agreement, the Partnership Act 1890 provides that all profits are shared equally, partners are jointly and severally liable for debts of the partnership and all partners can participate in management.

How well did you know this?
1
Not at all
2
3
4
5
Perfectly
4
Q

Which ONE of the following is correct?

A) Directors of a company can also be shareholders in the same company.

B) All companies are required to have one annual general meeting each year.

C) A private company can raise equity finance by offering to sell its shares to the public.

D) Private limited companies are required to have at least two directors.
Correct

A

A) Directors of a company can also be shareholders in the same company

(Correct. In private limited companies it is very common for the directors and shareholders to be the same people)

How well did you know this?
1
Not at all
2
3
4
5
Perfectly
5
Q

Jason, Kim and Sophia have set up a brand consultancy business (Branded). The business is doing well and they now are looking to expand. They have a number of family members and friends who are willing to invest in the business. They seek your advice as to whether to incorporate the business as a private limited company. Which of the following is correct?

A) A key advantage of incorporation is that the directors of the company are able to run the company without any input from the shareholders.

B) A key advantage of incorporation is that shareholders are able to invest in shares in the company with their liability being limited to the amount unpaid on their shares (if any).

C) An advantage of incorporating the business as a private limited company is that Branded will be able to offer shares to the public in order to increase the capital available to the business.

D) Branded can only be incorporated as a private limited company if it has at least £50,000 of share capital.

A

B) A key advantage of incorporation is that shareholders are able to invest in shares in the company with their liability being limited to the amount unpaid on their shares (if any).

How well did you know this?
1
Not at all
2
3
4
5
Perfectly
6
Q

Which ONE of the following is correct in relation to a listed company?

A) Any company can list its shares on the London Stock Exchange.

B) A listed company is a public limited company whose shares are listed on a regulated investment exchange such as the London Stock Exchange.

C) The requirements of CA 2006 do not apply to listed companies.

D) All public companies are listed.

A

B) A listed company is a public limited company whose shares are listed on a regulated investment exchange such as the London Stock Exchange.

How well did you know this?
1
Not at all
2
3
4
5
Perfectly
7
Q

Which ONE of the following is correct in relation to the company’s articles of association?

A) The articles take effect as a contract between the members themselves only.

B) The articles will always override the provisions of CA 2006.

C) The articles take effect as a contract between the directors and the shareholders.

D) The articles take effect as a contract between the company and its members and between the members themselves.

A

D) The articles take effect as a contract between the company and its members and between the members themselves.

How well did you know this?
1
Not at all
2
3
4
5
Perfectly
8
Q

Which one of the following correctly states the position with regards to provisions in the Memorandum restricting the objects of companies formed under the Companies Act 1985?

A) Any restrictions in the Memorandum of companies incorporated under the 1985 Act are no longer binding, since under the CA 2006, the Memorandum has no constitutional significance.

B) Any restrictions in the Memorandum of companies incorporated under the 1985 Act take effect as if they were contained in the Articles and are binding until the Articles are amended or new Articles adopted.

C) Any restrictions in the Memorandum of companies incorporated under the 1985 Act continue to bind the company and cannot be removed.

A

B) Any restrictions in the Memorandum of companies incorporated under the 1985 Act take effect as if they were contained in the Articles and are binding until the Articles are amended or new Articles adopted.

How well did you know this?
1
Not at all
2
3
4
5
Perfectly
9
Q

Which one of the following correctly describes the position of the objects clause (the clause setting out the purposes for which the company was formed) of companies incorporated under the Companies Act 2006?

A) Companies formed under CA 2006 have unrestricted objects, unless a specific restriction is inserted into the company’s articles.

B) Companies formed under CA 2006 have unrestricted objects. It is not possible to restrict the objects of a company formed under CA 2006.

C) All companies formed under CA 2006 will contain a provision in the Memorandum setting out the purposes for which the company was formed. This is known as an “objects clause”.

A

A) Companies formed under CA 2006 have unrestricted objects, unless a specific restriction is inserted into the company’s articles

(Correct. Section 31 CA 2006 confirms this)

How well did you know this?
1
Not at all
2
3
4
5
Perfectly
10
Q

Your client has purchased a shelf company which has been incorporated with Model Articles and seeks your advice as to how to effect a change of name of the company. Which one of the following is correct in relation to the procedure required to change the name of the company?

A) A special resolution of the shareholders is required.

B) The name may be changed by a board resolution of the directors.

C) An ordinary resolution of the shareholders is required.

D) It is not possible to change the name of a company.

A

A) A special resolution of the shareholders is required.

(Correct. See s 77(1) CA 2006)

How well did you know this?
1
Not at all
2
3
4
5
Perfectly
11
Q

When incorporating a company from scratch, which one of the following correctly states the documents that must be filed at Companies House?

A) The Memorandum, Articles of Association (unless Model Articles are used without amendment), fee and form IN01.

B) The Memorandum, Articles of Association (unless Model Articles are used without amendment), Certificate of Incorporation and fee.

C) The Articles of Association (unless Model Articles are used without amendment), fee and form IN01.

D) The Articles of Association (unless Model Articles are used without amendment), fee, Share Certificates and form IN01.

A

A) The Memorandum, Articles of Association (unless Model Articles are used without amendment), fee and form IN01.

How well did you know this?
1
Not at all
2
3
4
5
Perfectly
12
Q

When does a company become a legal entity?

A) From the date of incorporation as set out on the certificate of incorporation.

B) From the date on which the incorporation documents are filed at Companies House.

C) From the date on which the incorporation documents are received by Companies House (if this is different from the date of filing).

D) From the date on which the company is allocated a registered number.

A

A) From the date of incorporation as set out on the certificate of incorporation.

How well did you know this?
1
Not at all
2
3
4
5
Perfectly
13
Q

What is the ‘nominal value’ of a share?

A) The maximum subscription price for that share.

B) The minimum subscription price for that share.

C) The amount over and above £1 that the shareholder pays for the share on subscription.

D) The amount paid by the shareholder for the share at the time of purchase.

A

B) The minimum subscription price for that share.

How well did you know this?
1
Not at all
2
3
4
5
Perfectly
14
Q

Which one of the following would be a Person with Significant Control in relation to a company?

A) A shareholder who holds 25% of the voting share capital in the company, who is also a director.

B) A shareholder who holds 30% of the voting share capital of the company.

C) Any director.

A

B) A shareholder who holds 30% of the voting share capital of the company.

How well did you know this?
1
Not at all
2
3
4
5
Perfectly
15
Q

What is the ‘issued share capital’ of a company?

A) The total amount in value (nominal and premium) of all shares in issue at that time.

B) The total number of shares in issue at that time.

C) The total number of shares that the company is permitted to allot.

D) The total amount in value that has been paid up on all shares in issue at that time.

A

A) The total amount in value (nominal and premium) of all shares in issue at that time.

How well did you know this?
1
Not at all
2
3
4
5
Perfectly
16
Q

What is the meaning of “quorum” for a board or general meeting?

A) The minimum number of people that must be present for the meeting to be valid.

B) The notice period required for the meeting to be valid.

C) A representative of a shareholder who is unable to attend the meeting.

D) The number of attendees at a meeting of the board or shareholders.

A

A) The minimum number of people that must be present for the meeting to be valid.

How well did you know this?
1
Not at all
2
3
4
5
Perfectly
17
Q

Which ONE of the following is correct?

A) The chairman has a casting vote under MA 13, but this may be removed. The chairman is chosen by the shareholders.

B) All private limited companies must appoint a chairman.

C) The chairman has a casting vote under MA 13, but this may be removed. The chairman is chosen by the board of directors.

A

C) The chairman has a casting vote under MA 13, but this may be removed. The chairman is chosen by the board of directors.

How well did you know this?
1
Not at all
2
3
4
5
Perfectly
18
Q

Which of the following correctly sets out the thresholds for ordinary and special resolutions of the shareholders?

A) An ordinary resolution is passed by more than 50% of the votes. A special resolution is passed by more than 75% of the votes.

B) An ordinary resolution is passed by more than 50% of the votes. A special resolution is passed by 75% or more of the votes.

C) An ordinary resolution is passed by 50% or more of the votes. A special resolution is passed by 75% or more of the votes.

D) An ordinary resolution is passed by 75% or more of the votes. A special resolution is passed by more than 50% of the votes.

A

B) An ordinary resolution is passed by more than 50% of the votes. A special resolution is passed by 75% or more of the votes.

How well did you know this?
1
Not at all
2
3
4
5
Perfectly
19
Q

Major decisions affecting the company (such as the power to remove a director and the power to change the company’s name) will be taken by which ONE of the following?

A) The partners

B) The board of directors

C) A committee of directors.

D) The shareholders

A

D) The shareholders

(This is the correct answer. The shareholders take the important decisions and the examples given are reserved for them by CA 2006)

How well did you know this?
1
Not at all
2
3
4
5
Perfectly
20
Q

Bill, Paul, Simon and Ben are all shareholders of Magic Music Limited (‘Magic’). They have recently decided to rebrand Magic as they feel that the company’s name is outdated. As part of the rebranding they will need to change Magic’s name. They each hold the following number of shares in Magic:

Bill - 28 shares

Paul - 48 shares

Simon - 20 shares

Ben - 4 shares

Magic’s Articles of Association do not deal with changes to Magic’s name and Magic intends to deal with the change of name at a forthcoming GM. Which ONE of the following statements is correct?

A) On a poll the resolution could be passed if only Bill and Paul voted in favour of it.

B) On a poll the resolution could be passed if Bill, Simon and Ben all voted in favour of it.

C) On a show of hands all of the shareholders would need to vote in favour of the resolution in order for it to be passed.

D) If only Bill and Paul voted in favour of the resolution on a show of hands then it could be passed.

A

A) On a poll the resolution could be passed if only Bill and Paul voted in favour of it.

(This is the correct answer. Changing the name of the company requires a special resolution (s 77 CA 2006). A special resolution requires the approval of 75% or more of the votes. Bill and Paul together hold 76% of the shares so on a poll vote they can pass a special resolution)

How well did you know this?
1
Not at all
2
3
4
5
Perfectly
21
Q

Which of the following correctly represents the documents that must be filed at Companies House when a company votes to amend its articles?

A) Copy of the amended articles, copy of the GM and BM minutes.

B) Copy of the amended articles, copy of the GM minutes.

C) Copy of a special resolution, copy of the amended articles.

D) Copy of an ordinary resolution, copy of the amended articles.

A

C) Copy of a special resolution, copy of the amended articles.

(This is the correct answer. A special resolution is required in order to amend a company’s articles of association pursuant to s 21(1) CA 2006, not an ordinary resolution. All special resolutions must be filed at Companies House, together with the amended articles)

How well did you know this?
1
Not at all
2
3
4
5
Perfectly
22
Q

Which of the following best describes the key advantages of the company business model?

A) A company is a separate legal entity and the shareholders have limited liability.

B) Companies have more flexibility in raising finance since they may issue shares to raise money and can also provide security to lenders.

C) A company is a separate legal entity, shareholders have limited liability and there is more flexibility in raising finance since companies may issue shares to raise money and can also provide security to lenders.

D) A company is a separate legal entity and the shareholders have limited liability. There is no requirement for publicly filed accounts.

A

C) A company is a separate legal entity, shareholders have limited liability and there is more flexibility in raising finance since companies may issue shares to raise money and can also provide security to lenders.

How well did you know this?
1
Not at all
2
3
4
5
Perfectly
23
Q

Which of the following correctly sets out the business models in which participants are not generally liable for the debts or liabilities of the business?

A) Companies only

B) LLPs, partnerships and companies.

C) Partnerships and sole traders.

D) LLPs and companies

E) LLPs only.

A

D) LLPs and companies

How well did you know this?
1
Not at all
2
3
4
5
Perfectly
24
Q

Two women are setting up a business together. They are primarily concerned with their ability to raise finance and limiting their own personal liabilities as investors. They also do not want to spend lots of money setting up the business.

Which of the following is the best advice to the women in terms of the most suitable business model to use for their new business?

A) They should incorporate a private limited company. Whilst there are costs involved in setting up the company, they will enjoy limited personal liability and have a choice of finance options.

B) The women should incorporate an LLP. They will enjoy limited personal liability.

C) They should incorporate an LLP. Whilst there are costs involved in setting up the company, they will enjoy limited personal liability and be able to issue shares and take out bank loans in the LLP’s name.

D) They should trade as a traditional partnership. It is free to set up and the partnership has the ability to take out loans.

A

A) They should incorporate a private limited company. Whilst there are costs involved in setting up the company, they will enjoy limited personal liability and have a choice of finance options.

How well did you know this?
1
Not at all
2
3
4
5
Perfectly
25
Q

Which of the following correctly describes the structure and tax position of an LLP?

A) An LLP has a separate legal personality. The LLP pays income tax on its profits and capital gains tax on its gains.

B) An LLP has a separate legal personality. The partners are taxed as individuals on their shares of the profits and gains.

C) An LLP is not a separate entity from its partners. The LLP pays corporation tax on the partnership profits.

D) An LLP is not a separate entity from its partners. The partners are taxed as individuals on their shares of the profits and gains.

E) An LLP has a separate legal personality. The partners pay corporation tax on the partnership profits.

A

B) An LLP has a separate legal personality. The partners are taxed as individuals on their shares of the profits and gains.

How well did you know this?
1
Not at all
2
3
4
5
Perfectly
26
Q

Which one of the following correctly describes the structure and tax position of a traditional partnership?

A) A partnership is not a separate entity from its partners and the partners are taxed as individuals.

B) A partnership has a separate legal personality. The partnership pays corporation tax on the partnership profits.

C) A partnership has a separate legal personality and the partners are taxed as individuals.

D) A partnership has a separate legal personality. The partnership pays income tax on the partnership profits.

A

A) A partnership is not a separate entity from its partners and the partners are taxed as individuals.

(Correct. A partnership is not a separate entity from its partners. The partners are taxed as individuals and pay income tax on their share of the profits and capital gains tax on their share of the gains)

How well did you know this?
1
Not at all
2
3
4
5
Perfectly
27
Q

What is meant by “double taxation of profits” in the context of a company?

A) The company pays income tax on its profits before the payment of dividends to its shareholders. The individual shareholders who receive the dividends will pay income tax on the amount of the dividend.

B) The company pays corporation tax on its profits before the payment of dividends to its shareholders. The individual shareholders who receive the dividends will pay capital gains tax on the amount of the dividend.

C) The company pays capital gains tax on its profits before the payment of dividends to its shareholders. The individual shareholders who receive the dividends will pay income tax on the amount of the dividend.

D) The company pays corporation tax on its profits before the payment of dividends to its shareholders. The individual shareholders who receive the dividends will pay income tax on the amount of the dividend.

A

D) The company pays corporation tax on its profits before the payment of dividends to its shareholders. The individual shareholders who receive the dividends will pay income tax on the amount of the dividend.

How well did you know this?
1
Not at all
2
3
4
5
Perfectly
28
Q

X Ltd is converting to a Plc. Which one of the following correctly sets out when X Plc may commence trading as a Plc?

A) Once the trading certificate has been issued by the Registrar of companies.

B) Once the new certificate of incorporation has been issued by the Registrar of companies.

C) Once the directors have changed the name of the company at its registered office and on its stationery.

D) Once the new certificate of incorporation and trading certificate showing that the company’s allotted share capital is not less than the minimum has been issued by the Registrar of companies.

A

D) Once the new certificate of incorporation and trading certificate showing that the company’s allotted share capital is not less than the minimum has been issued by the Registrar of companies.

How well did you know this?
1
Not at all
2
3
4
5
Perfectly
29
Q

X Ltd is a private limited company. The board would like to re-register the company as a public limited company.

X Ltd currently has 1 director and 2 shareholders but does not have a company secretary. The company has an issued share capital of £100.

Which of the following statements about the structure of the re-registered PLC is correct?

A) The company will need at least one more shareholder.

B) Once re-registered as a PLC the company can continue to use written resolutions to make shareholder decisions.

C) The company will not need to appoint a company secretary but may do so if it wishes.

D) The company will not need to increase its share capital to be re-registered as a PLC.

E) The company will need to appoint at least one more director to the board.

A

E) The company will need to appoint at least one more director to the board.

How well did you know this?
1
Not at all
2
3
4
5
Perfectly
30
Q

ABC Plc is a public limited company. It has decided to seek listing on the London Stock Exchange. ABC Plc has a private limited subsidiary called DEF Ltd.

Which one of the following statements is correct?

A) DEF Ltd should apply for listing once ABC Plc has listed its shares.

B) ABC Plc will only be able to offer shares to the public once it is a listed company.

C) It is not the company ABC Plc that will be listed, but its shares.

D) As a Plc, ABC Plc must apply to have its shares listed.

E) The shares of DEF Ltd will also be listed once ABC Plc is listed.

A

C) It is not the company ABC Plc that will be listed, but its shares.

How well did you know this?
1
Not at all
2
3
4
5
Perfectly
31
Q

A shareholder of a private limited company has bought 1000 £1 shares in the company but has only paid the company £100 so far.

The company has become insolvent but owes a creditor £25,000. The creditor is threatening to bring proceedings for the whole amount against the shareholder.

Which one of the following statements is correct?

A) The company has limited liability to the amount invested by the shareholders.

B) The shareholder is jointly and severally liable with all the other shareholders to the creditor for the total sum of the claim.

C) The shareholder’s liability will be limited to a proportion of the amount claimed depending on the proportion of their shareholding.

D) The extent of the shareholder’s liability is a further payment of £1000 into the company. The shareholder will not be liable to the creditor for any further sums.

E) The extent of the shareholder’s liability is a further payment of £900 into the company. The shareholder will not be liable to the creditor for any further sums.

A

E) The extent of the shareholder’s liability is a further payment of £900 into the company. The shareholder will not be liable to the creditor for any further sums.

How well did you know this?
1
Not at all
2
3
4
5
Perfectly
32
Q

A man applied to Companies House two days ago to incorporate a company. The certificate of incorporation was issued today by Companies House. The man is the sole shareholder and director of the company.

Which one of the following statements about the legal personality of the company is correct?

A) If the director of the company changes the company is automatically wound up.

B) The company has a separate legal personality from today.

C) It is not possible for the company to have separate legal personality from the sole shareholder and director because they will always be seen as one entity.

D) The company has a separate legal personality from the date that the man applied to incorporate the company.

E) It will be possible for the company to own its own property or enter into contracts from the date of the man applied to incorporate the company.

A

B) The company has a separate legal personality from today.

How well did you know this?
1
Not at all
2
3
4
5
Perfectly
33
Q

The significance of limited liability to shareholders in a company may be overridden to some extent by which one of the following?

A) Contractual means such as banks requiring a guarantee from the shareholder(s) of the company as part of lending money to the company.

B) Where the company’s shareholder is itself a company, the parent or holding company will have unlimited liability for the debts of its subsidiary company.

C) Where the shareholders are also directors, they will have personal liability for debts of the company.

D) Where the company has only one shareholder, that shareholder will have unlimited liability.

A

A) Contractual means such as banks requiring a guarantee from the shareholder(s) of the company as part of lending money to the company.

How well did you know this?
1
Not at all
2
3
4
5
Perfectly
34
Q

A and B entered a contract 2 months ago. B has breached a term of the contract. The term was stated in the contract expressly to be a condition. What remedies are available to A?

A) It will be up to the Court to decide what remedy A has.

B) A must repudiate the contract since the term breached was a condition.

C) A can only sue for damages as the term breached was a condition not a warranty.

D) A can repudiate the contract and/or sue for damages.

E) A can claim liquidated damages because it is a breach of condition.

A

D) A can repudiate the contract and/or sue for damages.

How well did you know this?
1
Not at all
2
3
4
5
Perfectly
35
Q

X has made an offer to Y to enter into a contract on X’s standard terms of business.

Y sent a letter in the post two weeks ago accepting X’s offer in full. It was sent to the correct address. X has not yet received the acceptance. Can X revoke the offer?

A) Yes. X can revoke the offer because they have not yet received acceptance.

B) Yes. X can revoke the offer as long as Y receives the revocation letter before X receives the acceptance.

C) No. X cannot revoke the offer because it has been validly accepted by Y under the postal rule.

D) Yes. X can revoke the offer because the acceptance will lapse after a passage of time.

E) No. X cannot revoke the letter because it is not possible to revoke a written offer.

A

C) No. X cannot revoke the offer because it has been validly accepted by Y under the postal rule.

How well did you know this?
1
Not at all
2
3
4
5
Perfectly
36
Q

A company is preparing to execute a document as a deed. The company has four directors.

Which of the following statements represents the correct form of execution?

A) The document can be validly executed by one of the directors alone.

B) All of the directors will need to sign the document for it to be validly executed as a deed.

C) The document will only be validly executed if the company seal is attached.

D) The document can be validly executed if two directors sign.

E) The company will need a independent witness to sign the document alongside the signatories for it to be validly executed as a deed.

A

D) The document can be validly executed if two directors sign.

How well did you know this?
1
Not at all
2
3
4
5
Perfectly
37
Q

Ten partners are in a partnership without a written agreement. The partnership is a firm of accountants.

The partnership is not doing very well financially and is two months late with rent. The finance partner signed the lease agreement with the landlord five years ago on behalf of the partnership.

The senior partner will retire in two months’ time.

Which one of the following statements represents the correct position with regards to the liability of the partners for the overdue rent?

A) All ten of the partners are jointly liable for the rent.

B) The senior partner will not be liable for the overdue rent once they retire in two months.

C) The partnership is liable for the overdue rent.

D) Only the finance partner will be liable as they signed the lease.

E) All ten of the partners are jointly and severally liable for the rent.

A

A) All ten of the partners are jointly liable for the rent.

How well did you know this?
1
Not at all
2
3
4
5
Perfectly
38
Q

Which one of the following is correct in relation to the tax treatment of partnerships?

A) Partners in a partnership are liable to pay income tax and capital gains tax on their share of the income and capital gains of the partnership. The partnership itself is not liable to pay tax.

B) Partners in a partnership are not liable to any tax in relation to the profits and gains of the partnership. The partnership itself is liable to pay corporation tax.

C) Partners in a partnership are liable to pay income tax and capital gains tax on their share of the income and capital gains of the partnership. The partnership itself is also liable to pay income and capital gains tax.

D) Partners in a partnership are liable to pay income tax and capital gains tax on their share of the income and capital gains of the partnership. The partnership itself is also liable to pay corporation tax.

E) Partners in a partnership are liable to pay income tax and capital gains tax on their share of the income and capital gains of the partnership. Only the partners therefore need to submit tax returns.

A

A) Partners in a partnership are liable to pay income tax and capital gains tax on their share of the income and capital gains of the partnership. The partnership itself is not liable to pay tax

(Correct. A partnership is not a separate legal entity and therefore does not pay tax)

How well did you know this?
1
Not at all
2
3
4
5
Perfectly
39
Q

A firm of surveyors is seeking your advice in relation to a contract entered into by one of the partners. The contract was for a building project and the managing partner had agreed that the partners would do the work for £5,000. The remaining partners are upset as they believe that the managing partner significantly under-quoted for this project and that in fact they should be charging at least £7,000.

Work has not yet started on this contract since the remaining partners are seeking to avoid the contract on the basis that the managing partner was not authorised to enter into it. The written partnership agreement requires that all quotes for work should be signed off by at least two partners.

Which one of the following is the best advice to the partnership?

A) The partnership will be bound by the contract since the act is for carrying on business of the kind carried on by the firm, in the usual way. There does not appear to be anything on the facts to indicate that the customer knew that the managing partner was not authorised to enter into the contract on behalf of the firm.

B) The partnership will not be bound by the contract since the written partnership agreement requires that all quotes for work should be signed off by at least two partners. The managing partner was therefore acting in breach of the partnership agreement.

C) The partnership will be bound by the contract since the other partners, in allowing the managing partner to give the quote, will be said to have ratified the partner’s act.

A

A) The partnership will be bound by the contract since the act is for carrying on business of the kind carried on by the firm, in the usual way. There does not appear to be anything on the facts to indicate that the customer knew that the managing partner was not authorised to enter into the contract on behalf of the firm.

How well did you know this?
1
Not at all
2
3
4
5
Perfectly
40
Q

Three individuals (Partner X, Partner Y, and Partner Z) began trading together as a partnership 12 months ago. The partners have never signed a partnership agreement.

Partner X contributed 55% of the start up capital; partner Y contributed 40% of the start up capital and Partner C contributed 5% of the start up capital.

Which of the following statements represents the correct position with regards to the rights to the profits of the partnership and a salary for each partner under the default provisions of the Partnership Act 1890?

A) None of the partners are entitled to a share of the profits; all of the partners are entitled to a an equal salary.

B) All three partners are entitled to an equal share of the profits; none of the partners are entitled to a salary.

C) The three partners are entitled to a profit share in the proportion to their capital investments; and a salary in the proportion to their capital investments.

D) All three partners are entitled to an equal share of the profits; all three partners are entitled to receive a salary in the proportion of their capital investments.

E) The three partners are entitled to a profit share in the proportion to their capital investments; none of the partners are entitled to a salary.

A

B) All three partners are entitled to an equal share of the profits; none of the partners are entitled to a salary.

How well did you know this?
1
Not at all
2
3
4
5
Perfectly
41
Q

Three individuals have started a business together and have been working in partnership for six months. They have not entered into any formal partnership agreement. Two of the individuals would like to introduce a new partner to the business, but the third individual has reservations about the new proposed partner.

Which of the following is the correct advice as to the appointment of the new partner?

A) In the absence of agreement, the appointment of a new partner requires a majority vote therefore two of the partners can appoint the proposed new partner without the agreement of the third partner.

B) In the absence of agreement, the appointment of a new partner requires unanimity therefore two of the partners cannot appoint the proposed new partner without the agreement of the third existing partner.

C) In the absence of agreement, it is not possible to introduce a new partner to an existing partnership. The effect of this will be that the existing partnership is dissolved, and a new partnership will arise automatically once the new individual commences working with the other partners.

A

B) In the absence of agreement, the appointment of a new partner requires unanimity therefore two of the partners cannot appoint the proposed new partner without the agreement of the third existing partner.

How well did you know this?
1
Not at all
2
3
4
5
Perfectly
42
Q

Five partners are in a partnership with no formal partnership agreement. Four of the partners are unhappy with the fifth partner and wish to remove the partner.

Which of the following statements represents the correct advice to the partners about removal of the fifth partner?

A) It will be possible for the four partners to remove the fifth partner because they represent a majority.

B) Once removed, the partnership will continue without the fifth partner.

C) It will not be possible to remove the fifth partner without unanimous consent.

D) Once removed, the partnership will continue without the fifth partner, provided that the partners appoint a fifth partner to take the place of the expelled partner.

E) Once the partner leaves, they will have no claim on the partnership assets.

A

C) It will not be possible to remove the fifth partner without unanimous consent.

How well did you know this?
1
Not at all
2
3
4
5
Perfectly
43
Q

Two months ago a couple set up an LLP through which to operate their business. They had been trading as a traditional partnership before the incorporation of the LLP.

Which of the following best describes the change in taxation?

A) The LLP will pay corporation tax and the partners will continue to pay income tax.

B) The partners will pay income tax on their dividends from the LLP instead of their earning from the partnership.

C) There is no change in taxation; the LLP is treated as a partnership for taxation purposes.

D) The partners will now need to pay corporation tax, instead of income tax.

E) Both the LLP and the partners will pay income tax.

A

C) There is no change in taxation; the LLP is treated as a partnership for taxation purposes.

How well did you know this?
1
Not at all
2
3
4
5
Perfectly
44
Q

Two individuals want to start a business together and their main concern is to limit their personal liability. They are choosing between a traditional partnership and an LLP. They are both going to be active in the running of the business. They do not want to spend a lot of money in the set up of the business but they need the ability to raise some finance in the future.

Which of the following represents the best advice to this client for their business?

A) The individuals should incorporate an LLP because they will enjoy limited liability.

B) The individuals should incorporate an LLP because it is not necessary to have a partnership agreement, which will save costs.

C) The individuals should incorporate an LLP because it will be cheaper than starting a traditional partnership.

D) The individuals should trade as a partnership because it will be cheaper.

E) The individuals should trade as a partnership as they will have limited liability for debts.

A

A) The individuals should incorporate an LLP because they will enjoy limited liability.

How well did you know this?
1
Not at all
2
3
4
5
Perfectly
45
Q

Which one of the following is generally seen as an advantage of an LLP when compared to a private limited company?

A) LLPs are capable of creating a floating charge over the assets of the LLP.

B) Partners in an LLP have limited liability.

C) There is little set statutory procedure to follow for an LLP when the partners are making decisions and in day to day management.

D) LLPs are not required to file accounts at Companies House.

E) An LLP has a separate legal personality from its members.

A

C) There is little set statutory procedure to follow for an LLP when the partners are making decisions and in day to day management.

How well did you know this?
1
Not at all
2
3
4
5
Perfectly
46
Q

Two individuals (Partner A and Partner B) began trading together as a partnership five years ago. Two years ago a third partner, (Partner C) joined the partnership.

Partner A put in 75% of the start up capital and partner B put in the remaining 25%. Partner C has never contributed any capital but the partnership uses a warehouse owned by Partner C. The partners have never entered into any formal agreement.

Which of the following statements represents the correct position with regards to the rights to the profits of the partnership and a salary for each partner under the default provisions of the Partnership Act 1890?

A) Partner A and Partner B are entitled to an equal share of the profits, but Partner C is not entitled to any profits. None of the partners are entitled to a salary.

B) The three partners are entitled to a share of the profits equal to the percentage of their original capital investment and no salary.

C) Partner A and Partner B are entitled to an equal share of the profits, but Partner C is not entitled to any profits. All three partners are entitled to an equal salary.

D) The three partners are entitled to an equal share of the profits and a salary in equal proportions to their original capital investment.

E) The three partners are entitled to an equal share of the profits but none of the Partners are entitled to a salary

A

E) The three partners are entitled to an equal share of the profits but none of the Partners are entitled to a salary

(Correct. Under the Partnership Act 1890 all partners are entitled to an equal share of the profits, regardless of their original investment and the PA 1890 states that unless it is agreed to the contrary no partner shall take a salary)

How well did you know this?
1
Not at all
2
3
4
5
Perfectly
47
Q

Two individuals want to start a business together and are keen to limit their liability. They are both going to be active in the running of the business. They do not want to spend a lot of money in the set up of the business, but they need the ability to raise some finance in the future in the business name.

Which of the following would be the best vehicle for their business?

A) A partnership

B) A sole trader

C) A public limited company

D) A limited liability partnership

E) A private limited company

A

E) A private limited company

(Correct. This option has the ability to raise finance and limit the personal liability of the individuals)

How well did you know this?
1
Not at all
2
3
4
5
Perfectly
48
Q

You act for a partnership which is made up of 8 partners. There is no written partnership agreement. Profits have always been shared equally between the partners.

Three years ago, the partners all agreed to take out a loan to renovate their main office. The partners all contributed equally to repaying the loan. Unfortunately, the partnership has not been profitable, and they have recently defaulted on the loan repayments.

One of the partners is about to retire. No documentation has been drafted to confirm the details of her retirement. Can the partner who is about to retire be liable for repaying any of the loan?

A) Yes, because there is no written partnership agreement therefore under the default provisions of the Partnership Act 1890, all those persons who were partners at the time that the loan agreement was entered into will be jointly liable for repaying the loan regardless of retirement.

B) No, because there is no written partnership agreement therefore under the default provisions of the Partnership Act 1890, only the current partners will be jointly liable for debts of the partnership.

C) No, because the partners have not entered into a written partnership agreement which deals with liability on retirement therefore once the partner retires, she is no longer a partner and therefore has no liability for any debts of the partnership.

D) No, because there is no written partnership agreement therefore under the default provisions of the Partnership Act 1890, only the current partners will be jointly and severally liable for debts of the partnership.

E) Yes, because there is no written partnership agreement therefore under the default provisions of the Partnership Act 1890, all those persons who were partners at the time that the loan agreement was entered into will be be jointly and severally liable for repaying the loan regardless of retirement.

A

A) Yes, because there is no written partnership agreement therefore under the default provisions of the Partnership Act 1890, all those persons who were partners at the time that the loan agreement was entered into will be jointly liable for repaying the loan regardless of retirement.

(Correct. Under the Partnership Act 1890 every partner is jointly liable for contractual debts and a partner will still be liable even though they have retired)

How well did you know this?
1
Not at all
2
3
4
5
Perfectly
49
Q

Client A and client B are looking to start up a new business offering commercial cleaning services locally. The only requirements of the clients are that (i) the profits and capital of the business are split equally between them and (ii) their liability is limited to their investment in the business (the ‘Agreed Terms’). Provided that these requirements are met, the clients want as little formality and documentation as possible.

What type of partnership would be most appropriate for the clients?

A) A partnership because although it requires a written agreement to operate on the Agreed Terms and is not a separate legal entity, it does offer limited liability.

B) A limited liability partnership because although it requires a written agreement to operate on the Agreed Terms, it is a separate legal entity offering limited liability.

C) A limited liability partnership because although it is not a separate legal entity it does offer limited liability and does not require a written agreement to operate on the Agreed Terms.

D) A limited liability partnership because it is a separate legal entity offering limited liability and does not require a written agreement to operate on the Agreed Terms.

E) A partnership because although it is not a separate legal entity, it offers limited liability and does not require a written agreement to operate on the Agreed Terms.

A

D) A limited liability partnership because it is a separate legal entity offering limited liability and does not require a written agreement to operate on the Agreed Terms

(Correct. Whilst the other answer options might sound plausible, they are each incorrect. A partnership is not a separate legal entity and does not offer limited liability for the partners whereas the opposite is true for a limited liability partnership. Additionally, a partnership agreement is not legally required to set up a partnership or limited liability partnership and would not strictly be needed if the terms agreed matched the statutory default provisions)

How well did you know this?
1
Not at all
2
3
4
5
Perfectly
50
Q

Your client is in the process of setting up an online recruitment business. In the future, it is hoped that staff will be employed to undertake a variety of tasks within the business, but for now, all work is carried out by your client who has secured investment from a family member. It has been agreed verbally that the family member will not be entitled salary and will not have any involvement in the day-to-day running of the business. Your client intends to take a salary from the business, but he has not discussed this with the family investor.

You have advised your client on the choice of business medium for this venture and your client has decided that a partnership would be the best option. Your client is keen to keep legal work and formality to a minimum at this stage and has asked you to explain the implications of continuing without a partnership agreement until the business has a regular turnover.

Which of the following statements best describes the impact of your client accepting the investment and continuing without a partnership agreement?

A) Neither partner would be entitled to any salary, both partners would be entitled to equal shares in the profits of the partnership and decisions would require the consent of both partners.

B) Both partners would be entitled to equal salaries, equal shares in the profits of the partnership and decisions would require the consent of both partners.

C) Neither partner would be entitled to any salary, your client would be entitled to all of the profits of the partnership and decisions would be made by your client alone.

D) Neither partner would be entitled to any salary, both partners would be entitled to equal shares in the profits of the partnership and decisions would be made by your client alone.

E) Both partners would be entitled to equal salaries and equal shares in the profits of the partnership, but decisions would be made by your client alone.

A

D) Neither partner would be entitled to any salary, both partners would be entitled to equal shares in the profits of the partnership and decisions would be made by your client alone

(Correct. The Partnership Act 1890 contains a default code, which applies to relations between the partners themselves in the absence of any contrary agreement. Here, there is a contrary agreement which provides that the family member will not be entitled to a salary and will not participate in decisions of the partnership. The default provisions will apply in respect of your client’s salary (no entitlement) and profit share (equal shares))

How well did you know this?
1
Not at all
2
3
4
5
Perfectly
51
Q

You are advising a client about incorporating a limited company.

Which of the following statement about the constitution of the company is correct?

A) The client will not need a memorandum of association to incorporate a company.

B) The newly incorporated company will have unrestricted objects unless the client chooses to restrict them.

C) The main constitutional documents will be the memorandum.

D) If there is a conflict between the Articles of Association and the CA 2006, the Articles of Association will prevail.

A

B) The newly incorporated company will have unrestricted objects unless the client chooses to restrict them.

How well did you know this?
1
Not at all
2
3
4
5
Perfectly
52
Q

A company was incorporated in 2010. Which of the following statements about the company’s memorandum is correct?

A) The memorandum has no constitutional significance but it is required to be filed on incorporation. It simply amounts to a declaration on the part of the company’s subscribers that they wish to form a company and agree to become members of that company.

B) The memorandum is a constitutional document. It will include an objects clause setting out the purposes for which the company has been formed. Any acts outside the scope of the objects clause are said to be ultra vires.

C) Since this company was incorporated in 2010, it would not have required a memorandum.

A

A) The memorandum has no constitutional significance but it is required to be filed on incorporation. It simply amounts to a declaration on the part of the company’s subscribers that they wish to form a company and agree to become members of that company.

How well did you know this?
1
Not at all
2
3
4
5
Perfectly
53
Q

Which of the following statements correctly describes the legal effect of the articles of a company?

A) The Articles take effect as a contract between the members themselves in their personal capacities and therefore members may enforce provisions contained in the Articles directly against other members.

B) The Articles take effect as a contract between the members themselves in their personal capacities. Members are able to enforce the provisions of the Articles both against the company and directly against other members.

C) The Articles take effect as a contract between the company and its members. Members are able to enforce the provisions of the Articles both against the company and directly against other members.

D) The Articles take effect as a contract between the company and its members. Members will only be able to enforce provisions contained in Articles against other members through the company itself.

A

D) The Articles take effect as a contract between the company and its members. Members will only be able to enforce provisions contained in Articles against other members through the company itself.

How well did you know this?
1
Not at all
2
3
4
5
Perfectly
54
Q

You are advising a client on a shelf company conversion. As part of the conversion, the client wishes to change the name of the company. The company has unamended Model Articles.

Which one of the following statements about the company name is correct?

A) The new name will be effective once the Registrar of Companies issues a new certificate of incorporation.

B) The company requires an ordinary resolution to change the name of the company.

C) The new name will be effective once the Company votes to change the name.

D) The new name will be effective once the correct form has been submitted to Companies House.

E) The Directors can change the name themselves once they are appointed to the Board.

A

A) The new name will be effective once the Registrar of Companies issues a new certificate of incorporation.

How well did you know this?
1
Not at all
2
3
4
5
Perfectly
55
Q

You are instructed to convert a shelf company PP123 Limited for your client. The one issued share will be transferred to your client WYX Plc and your client plans to appoint 2 directors from its own board to the board of the shelf company.

Which one of the following statements about the conversion is correct?

A) Your client will control PP123 Limited once it is entered on the register of members.

B) PP123 Limited will be a separate legal entity after the conversion.

C) Your client will control PP123 Limited once its shareholding is paid for.

D) The current directors should resign before the new directors are appointed to ensure a clean break.

E) Your client will control PP123 Limited once the share transfer is registered at Companies House.

A

A) Your client will control PP123 Limited once it is entered on the register of members.

How well did you know this?
1
Not at all
2
3
4
5
Perfectly
56
Q

Your client wishes to incorporate from scratch a new company, which will have unamended Model Articles. Which one of the following correctly describes the documents that will need to be filed at Companies House?

A) The memorandum, fee, the company’s Articles and Form IN01.

B) The memorandum, fee and Form IN01.

C) The fee, Model Articles and Form IN01.

D) The memorandum, fee, Model Articles and Form IN01.

E) The fee and Form IN01.

A

B) The memorandum, fee and Form IN01.

How well did you know this?
1
Not at all
2
3
4
5
Perfectly
57
Q

A company has unamended model articles and 4 shareholders with the following shareholdings:

A - 500 (50%)

B - 250 (25%)

C - 150 (15%)

D - 100 (10%)

The company wishes to shareholders to pass a special resolution using the written resolution procedure.

Which of the following statements is correct advice regarding the procedure?

A) A and B can pass the special resolution alone.

B) If C and D abstain, their votes won’t count.

C) All shareholders must unanimously consent to the written resolution.

D) If one of the shareholders doesn’t vote for 21 days because they miss the email, it will be too late to vote for the resolution as the written resolution will have lapsed.

E) The special resolution can only be passed if 3 of the 4 shareholders vote in favour.

A

A) A and B can pass the special resolution alone.

How well did you know this?
1
Not at all
2
3
4
5
Perfectly
58
Q

A company has unamended model articles and 4 shareholders with the following shareholdings:

A - 500 (50%)

B - 250 (25%)

C - 150 (15%)

D - 100 (10%)

The company wishes to shareholders to pass an ordinary resolution at a General Meeting. All shareholders will attend the meeting and vote.

Which of the following statements is correct advice regarding the procedure?

A) A can pass the ordinary resolution alone.

B) If A is not in favour of the resolution, it cannot be passed.

C) The ordinary resolution can only be passed if 3 of the 4 shareholders vote in favour.

D) If B, C and D all vote in favour of the ordinary resolution, it will be passed despite A voting against it.

A

B) If A is not in favour of the resolution, it cannot be passed

(Correct. A holds 50% of the shares therefore can call a poll vote and is able to block the ordinary resolution (since more than 50% is required to pass))

How well did you know this?
1
Not at all
2
3
4
5
Perfectly
59
Q

A company has unamended model articles and 4 shareholders with the following shareholdings:

A - 500 (50%)

B - 250 (25%)

C - 150 (15%)

D - 100 (10%)

The company wishes to shareholders to pass a special resolution at a General Meeting. All shareholders will attend the meeting and vote.

Which of the following statements is the correct advice regarding the procedure?

A) If B votes against the special resolution, it cannot be passed.

B) A, B and either C or D will need to vote in favour for the special resolution to be passed.

C) The special resolution may be passed by A and B alone.

D) The special resolution can be passed if any 3 of the 4 shareholders vote in favour.

A

C) The special resolution may be passed by A and B alone

(Correct. A majority of 75% or more is required to pass a special resolution)

How well did you know this?
1
Not at all
2
3
4
5
Perfectly
60
Q

A company has unamended Model Articles and an issued share capital of 100,000 ordinary shares which is held by the following shareholders:

A 5,000 (5%)

B 20,000 (20%)

C 70,000 (70%)

D 2,000 (2%)

E 3,000 (3%)

Which one of the following combinations of shareholders would be able to consent to the meeting being held on short notice?

A) C, D and E

B) A, D and E

C) B and C

D) B, C and D

E) A, B, and D

A

D) B, C and D

How well did you know this?
1
Not at all
2
3
4
5
Perfectly
61
Q

A company with unamended Model Articles wishes to change its articles by special resolution. The company has four shareholders, two of whom are also directors. The company does not use the written resolution procedure and therefore a General Meeting will need to be held. Which of the following correctly sets out the requirements for a valid General Meeting?

A) The Board must call the General Meeting, giving 14 clear days’ notice. The quorum for the meeting is 2 shareholders.

B) The Board must call the General Meeting on reasonable notice. The quorum for the meeting is 2 shareholders.

C) The Board must call the General Meeting, giving 14 days’ notice. The quorum for the meeting is 2 shareholders.

D) The Board must call the General Meeting, giving 14 clear days’ notice. The quorum for the meeting is 1 shareholder. Voting takes place on a show of hands unless a poll vote is demanded.

E) The shareholders must call the General Meeting, giving 14 clear days’ notice. The quorum for the meeting is 2 shareholders. Voting takes place on a show of hands unless a poll vote is demanded.

A

A) The Board must call the General Meeting, giving 14 clear days’ notice. The quorum for the meeting is 2 shareholders.

How well did you know this?
1
Not at all
2
3
4
5
Perfectly
62
Q

A company has held a general meeting to:

Amend its articles by special resolution and
Authorise a loan to a director by ordinary resolution
Which one of the following options represents the correct pair of documents that must be filed at Companies House?

A) Copy of the special resolution and a copy of the ordinary resolution.

B) Copy of the amended articles and the general meeting minutes where the approval for the loan occurred.

C) Copy of the amended articles and a copy of the ordinary resolution.

D) Copy of special resolution and a copy of the loan agreement.

E) Copy of the amended articles and a copy of the special resolution.

A

E) Copy of the amended articles and a copy of the special resolution.

How well did you know this?
1
Not at all
2
3
4
5
Perfectly
63
Q

A corporate client of your law firm has bought a shelf company named Shelfco 123 Ltd (‘Shelfco Ltd’) on your recommendation. Shelfco Ltd has unamended Model Articles for a private company limited by shares. The Board of Directors of Shelfco Ltd wishes to change the name of the company to a more suitable commercial name. What is the correct procedure for changing the company name of Shelfco Ltd under the Companies Act 2006? (Assume the name chosen by the client is available and not subject to any objections by another party.)

A) To change the name of Shelfco Ltd under the Companies Act 2006, Shelfco Ltd’s shareholders must pass an Ordinary Resolution.

B) To change the name of Shelfco Ltd under the Companies Act 2006, Shelfco Ltd’s shareholders may pass an Ordinary Resolution. Alternatively, the Board may pass a Board Resolution.

C) To change the name of Shelfco Ltd under the Companies Act 2006, Shelfco Ltd’s shareholders may pass a special resolution. Alternatively, the Board may pass a Board Resolution.

D) To change the name of Shelfco Ltd under the Companies Act 2006, Shelfco Ltd’s Board merely needs to pass a Board Resolution.

E) To change the name of Shelfco Ltd under the Companies Act 2006, Shelfco Ltd’s shareholders must pass a special resolution.

A

E) To change the name of Shelfco Ltd under the Companies Act 2006, Shelfco Ltd’s shareholders must pass a special resolution.

(Correct. Section 77(1)(a) Companies Act 2006 requires the shareholders of Shelfco Ltd to pass a Special Resolution. Section 77(1)(b) does allow for the company to determine another method for changing the name in its articles but since Shelfco Ltd has unamended Model Articles, the only option is to use the special resolution procedure)

How well did you know this?
1
Not at all
2
3
4
5
Perfectly
64
Q

A company entered into a contract with an office equipment supplier to purchase 3 projectors. The contract was signed by the sole director on behalf of the company. The director and his wife are the shareholders of the company. The supplier delivered the projectors as agreed but the company failed to pay the purchase price.

Which statement best describes what legal action the supplier can take?

A) The supplier can sue the sole director for the purchase price.

B) The supplier can sue the company for the purchase price.

C) The supplier can sue the company and the shareholders for the purchase price.

D) The supplier can sue the shareholders for the purchase price.

E) The supplier can sue the company and the director for the purchase price.

A

B) The supplier can sue the company for the purchase price

(This is correct. The doctrine of separate legal liability means that the company is liable for its own debts)

How well did you know this?
1
Not at all
2
3
4
5
Perfectly
65
Q

Two individuals (A and B) want to incorporate a private limited company as soon as possible. A and B propose to each take 50% of the shares and become directors of the company. A is negotiating a supply agreement, on behalf of the not yet incorporated company with a company (C) to take effect once the company is incorporated. If A were to sign the agreement with C now, before the company is incorporated, who would be liable under the agreement?

A) A, B and the not yet incorporated company

B) A and B

C) A

D) Nobody, the contract would be void

E) The not yet incorporated company, once it is incorporated

A

C) A

(Correct. Under s 51(1) Companies Act 2006, the person signing the purported agreement between the unincorporated company and C would be personally liable)

66
Q

You are in the process of tailoring a shelf company (the “Company”) to meet a client’s requirements. The Company has adopted the unamended Model Articles of Association for Private Companies Limited by Shares. The client requires the Company’s name to be changed, the existing directors of the Company (“Existing Directors”) to be replaced with members of its team (the “New Directors”) and for the registered office of the Company to be changed before it is transferred to the client.

What board and shareholder resolutions are required to implement the client’s instructions most expeditiously (NOT including any resolutions required to convene meetings)?

A) A special resolution to change the Company’s name, board resolutions to appoint the New Directors, board resolutions to accept the resignations of the Existing Directors and a board resolution to change the Company’s registered office.

B) A special resolution to change the company’s name, ordinary resolutions to appoint the New Directors, ordinary resolutions to remove the Existing Directors and a board resolution to change the Company’s registered office.

C) An ordinary resolution to change the Company’s name, ordinary resolutions to appoint the New Directors, board resolutions to accept the resignations of the Existing Directors and an ordinary resolution to change the Company’s registered office.

D) A special resolution to change the Company’s name, board resolutions to appoint the New Directors, ordinary resolutions to remove the Existing Directors and a board resolution to change the Company’s registered office.

E) A special resolution to change the company’s name, board resolutions to appoint the New Directors, board resolutions to accept the resignations of the Existing Directors and a special resolution to change the Company’s registered office.

A

A) A special resolution to change the Company’s name, board resolutions to appoint the New Directors, board resolutions to accept the resignations of the Existing Directors and a board resolution to change the Company’s registered office

(Correct. This answer reflects the most expeditious and correct way of implementing the client’s required changes. While the other answer options might sound plausible, they are each incorrect. Since the Company has unamended Model Articles of Association for Private Companies Limited by Shares, a special resolution is required to change its name. Although it is possible to appoint directors by board resolution or by ordinary resolution, the most expeditious way of appointing the New Directors (and what is generally done in practice) is by board resolution. The Existing Directors would resign as directors and a board resolution would be passed accepting their letters of resignation. Changing the registered office of the Company would be effected by the passing of a board resolution, followed by the filing of the relevant form at Companies House)

67
Q

You act for a private limited company that was incorporated last year. The company’s only asset is the company bank account which holds £10,000 on deposit. The company has asked for your advice on changing its current company name.

Which one of the following statements is correct in relation to the company’s change of name?

A) The company will not be issued with a new certificate of incorporation following the change of name. The Registrar of Companies will change the company’s name online at Companies House only. The change of name is effective once the Registrar of Companies has received notice of the relevant special resolution.

B) The change of name will be effective once the Registrar of Companies has received notice of the relevant special resolution.

C) The company will be issued with a new certificate of incorporation following the change of name which will confirm its new name and new company number. The change of name is effective once this new certificate has been issued by the Registrar of Companies.

D) The change of name will be effective once the Registrar of Companies has issued the certificate of incorporation on a change of name.

E) The change of name will be effective as soon as the company has passed the required special resolution to change the company’s name.

A

D) The change of name will be effective once the Registrar of Companies has issued the certificate of incorporation on a change of name

(Correct. A change of name will be effective once the Registrar of Companies has issued the certificate of incorporation on a change of name (s 16 CA 2006). The company registration number will not change)

68
Q

A company has taken advice from an insolvency practitioner who is a qualified accountant about the financial position. Everybody on the board agrees with the advice and unanimously vote to follow it. The insolvency practitioner knows one of the directors socially.

Which one of the following statements reflects the insolvency practitioner’s position within the company?

A) It is likely that the insolvency practitioner will be considered a non-executive director.

B) It is likely that the insolvency practitioner will in effect be an alternate director.

C) The insolvency practitioner is unlikely to be considered a shadow director.

D) It is likely that the insolvency practitioner will be considered a shadow director as they have a social connection to one of the directors.

E) It is likely that the insolvency practitioner will be considered a shadow director as the board is unanimously following their advice.

A

C) The insolvency practitioner is unlikely to be considered a shadow director.

69
Q

A company which imports fine wines has been taking advice from the brother of one of the directors, who owns a vineyard in the South of France. The brother has been involved in the company’s business since its inception three years ago and has given advice both on sourcing the best wines but also on marketing and branding. Everybody on the board agrees with the advice and always unanimously votes to follow it.

Which one of the following statements reflects the brother’s position within the company?

A) It is likely that the brother will be considered a non-executive director of the company.

B) It is likely that the brother will be considered to be a de jure director.

C) It is likely that the brother will be considered to be an alternate director.

D) It is unlikely that the brother will be considered a shadow director as he has not been appointed as a director of the company.

E) The brother is likely to be considered a shadow director.

A

E) The brother is likely to be considered a shadow director.

(Correct. Section 251(1) CA 2006 defines a shadow director as ‘a person in accordance with whose directions or instructions the directors of the company are accustomed to act’)

70
Q

Which one of the following statements is correct in relation to the requirements for directors and a company secretary for public and private companies?

A) A private limited company must have a minimum of two directors. A secretary is not required. A public limited company must have a minimum of two directors and a company secretary.

B) A private limited company must have a minimum of one director. A secretary is not required. A public limited company must have a minimum of one director and a company secretary.

C) A private limited company must have a minimum of one director. A public limited company must have a minimum of two directors. A secretary is not required for any type of company, but most public limited companies do have company secretaries.

D) A private limited company must have a minimum of one director and a company secretary. A public limited company must have a minimum of two directors and a company secretary.

E) A private limited company must have a minimum of one director. A secretary is not required. A public limited company must have a minimum of two directors and a company secretary.

A

E) A private limited company must have a minimum of one director. A secretary is not required. A public limited company must have a minimum of two directors and a company secretary.

71
Q

ABC Ltd has just appointed a director to the Board by a board resolution. The new director will be employee of the company in the position of finance director and has been given a 1 year service contract in the first instance.

Which of the following statements is correct regarding the information that the company must disclose in relation to the new appointment?

A) Because their service is only for one year, the company is under no obligation to inform Companies House of the director’s appointment. The directors’ service contract must be kept for inspection at the company’s registered office.

B) The company must file form AP01 at Companies House in relation to the director’s appointment. The director’s service contract must also be filed at Companies House.

C) The company must file form AP01 at Companies House in relation to the director’s appointment. Because the service contract is for one year only, it will not need to be kept for inspection.

D) The company must file form AP01 at Companies House in relation to the director’s appointment. The directors’ service contract must be kept for inspection at the company’s registered office.

E) The company must file form AP01 at Companies House in relation to the director’s appointment. The new director will have to provide their personal address for public inspection.

A

D) The company must file form AP01 at Companies House in relation to the director’s appointment. The directors’ service contract must be kept for inspection at the company’s registered office.

72
Q

Which of the following statements correctly lists all the details concerning directors that must be included in the company’s annual accounts?

A) Information concerning directors’ salaries, bonus payments and pension entitlements and any compensation paid to directors and past directors for loss of office.

B) Information concerning directors’ salaries, bonus payments and pension entitlements.

C) Information concerning any compensation paid to directors and past directors for loss of office.

D) Information concerning directors’ salaries and bonus payments and any compensation paid to directors and past directors for loss of office.

A

A) Information concerning directors’ salaries, bonus payments and pension entitlements and any compensation paid to directors and past directors for loss of office.

73
Q

XYZ Ltd is a small private company with three directors who are all shareholders in the company. There is a fourth shareholder who is not a director. XYZ Ltd has unamended model articles.

XYZ Ltd wants to appoint a new director to the board. Which of the following statements represents the best advice to the company?

A) The appointment of the director has to be approved by an ordinary resolution of the shareholders.

B) It is not possible for the directors to appoint another director because there is a fourth shareholder who is not a director.

C) The directors can appoint the new director to the board with a unanimous decision.

D) The directors can appoint the new director to the board with a board resolution.

E) The appointment of the director has to be approved by a special resolution of the shareholders.

A

D) The directors can appoint the new director to the board with a board resolution.

74
Q

A company is proposing to build a sewage plant in a small town in the south of England. This sewage plant will lead to a massive increase in profit for the company and boost employment in the area.

Which of the following statements best describes the directors’ duty under s 172 CA 2006?

A) The boost in employment cannot be a factor.

B) If the decision promotes the short term profits of the company, the directors can still proceed with the decision.

C) One of the factors for the directors to consider in deciding whether to go ahead with the proposal will be the environmental impact of the plant.

D) As long as the minutes reflect the fact that s 172 CA 2006 has been considered the directors will not be in breach of their duty.

E) It will be impossible for the directors to comply with s 172 and build the sewage plant because the environmental considerations will outweigh the duty to promote the success of the company.

A

C) One of the factors for the directors to consider in deciding whether to go ahead with the proposal will be the environmental impact of the plant.

75
Q

Which of the following statements correctly describes the duty of directors under s 171 CA 2006?

A) Directors must act within the company’s constitution and must exercise their powers for the purposes for which they are conferred.

B) Directors must act within the company’s constitution and must exercise reasonable skill and care.

C) Directors must act within the company’s constitution and must exercise independent judgment.

D) Directors must promote the success of the company.

E) Directors must exercise their powers for the purposes for which they are conferred and must exercise reasonable skill and care.

A

A) Directors must act within the company’s constitution and must exercise their powers for the purposes for which they are conferred.

76
Q

To whom do directors owe their duties under s 171 – 177 CA 2006?

A) The shareholders.

B) The creditors of the company.

C) The company and the shareholders.

D) The shareholders and the creditors.

E) The company.

A

E) The company.

77
Q

One shareholder of XYX Ltd thinks that the financial director has breached their duty to the company under s 174 CA 2006 of exercising their reasonable care skill and diligence by not properly conducting due diligence on a recent large purchase of land which has cost the company thousands of pounds.

Which of the following statements is correct about the remedies for this breach if it is found to be a breach?

A) The shareholder will be entitled to claim any remedy for breach of fiduciary duty.

B) A breach of s 174 CA 2006 is not actionable by either the company of the shareholder.

C) The company will likely be entitled to damages for the breach.

D) The Company will likely be entitled to an order setting aside the transaction.

E) The shareholder will likely be entitled to damages for the breach.

A

C) The company will likely be entitled to damages for the breach.

78
Q

ABC Ltd wants to enter into a contract with DEF Ltd for DEF Ltd to supply it with £200,000 of office furniture. One of ABC Ltd’s directors holds 25% of the shares in DEF Ltd. ABC Ltd has unamended model articles.

Which of the following statements represents the best advice to the director with regard to their directors’ duties under the CA 2006?

A) Section 175 CA applies to the conflict and the director needs to avoid this conflict.

B) The director must declare their interest in the proposed transaction in writing.

C) As long as the director declares their interest, they will be able to vote on the transaction.

D) The director should declare their interest in the proposed transaction to the board of DEF Ltd at the earliest opportunity and before the transaction is interested into.

E) The director should declare their interest in the proposed transaction to the board of ABC Ltd at the earliest opportunity and before the transaction is interested into.

A

E) The director should declare their interest in the proposed transaction to the board of ABC Ltd at the earliest opportunity and before the transaction is interested into.

79
Q

In which one of the following situations would a director NOT need to declare their interest under s 177 CA 2006?

A) Where the board is voting on a transaction with another company in which the director has a small (5%) shareholding.

B) Where the board is voting on the terms of the director’s service contract.

C) Where the board is voting on the purchase of equipment worth £2,000 from the director’s father.

D) Where the board is considering the purchase of a property worth £300,000 from the sister of one of the directors.

A

B) Where the board is voting on the terms of the director’s service contract

(Correct. There is an exception under s 177(6)(c ) that applies here)

80
Q

A company wants to renew one of its directors’ service contracts. The company has one shareholder, which is its parent company.

The draft contract is for an initial period of two years. The director has the option to extend the contract for a further year, provided they give notice to the company six months before the expiry of the initial period. The company does not have the same right to renew. During the period of the agreement the company can only terminate the employment if the director breaches the disciplinary policy.

Which of the following statements is correct in respect of the need for shareholder approval of this contract?

A) The quickest the shareholder approval can be obtained is 15 days.

B) The company is a wholly owned subsidiary so is exempt from obtaining approval.

C) The company is renewing the service contract as the director is already employed, so no shareholder approval is necessary.

D) The shareholders of the parent company will need to give permission because they are the ultimate owners of the group.

E) Shareholder approval will be required by ordinary resolution because this contract fits within the definition of a guaranteed term of more than 2 years.

A

B) The company is a wholly owned subsidiary so is exempt from obtaining approval.

81
Q

Which one of the following service contracts falls within the definition of a long-term service contract under s 188?

A) A contract with a fixed term of three years. During the period of the agreement the company can terminate the employment on six months’ notice.

B) A contract with a fixed term of two years. During the period of the agreement the company can only terminate the employment if the director breaches the disciplinary policy.

C) A contract with an initial term of 18 months, where the director has an option to extend the contract for a further year, provided they give notice to the company six months before the expiry of the initial period. The company does not have the same right to renew. During the period of the agreement the company can only terminate the employment if the director breaches the disciplinary policy.

A

C) A contract with an initial term of 18 months, where the director has an option to extend the contract for a further year, provided they give notice to the company six months before the expiry of the initial period. The company does not have the same right to renew. During the period of the agreement the company can only terminate the employment if the director breaches the disciplinary policy.

(Not 3yrs as company can terminate contract on 6mos notice)

82
Q

A company wants to renew one of its director’s service contracts. The company has three shareholders, all of whom are also directors.

The draft contract is for an initial period of 18 months. The director has the option to extend the contract for a further year, provided they give notice to the company six months before the expiry of the initial period. The company does not have the same right to renew. During the period of the agreement the company can only terminate the employment if the director breaches the disciplinary policy.

Which of the following statements is correct in respect of the need for shareholder approval of this contract?

A) The company is renewing the service contract as the director is already employed, so no shareholder approval is necessary.

B) Shareholder approval will be required by ordinary resolution because this contract fits within the definition of a guaranteed term of more than two years.

C) If shareholder approval is necessary, the quickest it could be obtained is 15 days.

D) Since all of the shareholders of this company are also directors, no shareholder approval is necessary.

E) This contract does not fall into the definition of a long-term service contract since the guaranteed term is not over two years, therefore no shareholder approval is necessary.

A

B) Shareholder approval will be required by ordinary resolution because this contract fits within the definition of a guaranteed term of more than two years.

83
Q

ABC Ltd is proposing to buy a piece of land from the aunt of one of its directors. ABC Ltd has three shareholders including the director.

The land has been independently valued at £150,000 and all parties are happy with this valuation. The net asset value of ABC Ltd is £3.5 million.

Is the approval of the shareholders of ABC Ltd required for this transaction?

A) Shareholder approval is not required and therefore the director has no conflict of interest.

B) Shareholder approval is not required because “aunt” is not a person connected to a director and therefore the acquisition does not fall within the legislation.

C) The asset is not substantial in value because it is not more than 10% of the net asset value of the ABC Ltd.

D) The director will be precluded from voting at the General Meeting when the loan comes to be voted on because of MA 14.

E) Shareholder approval is required because the company is buying an asset from a person connected to a director.

A

B) Shareholder approval is not required because “aunt” is not a person connected to a director and therefore the acquisition does not fall within the legislation.

84
Q

ABC Ltd is proposing to buy some equipment from the father of one of its directors. ABC Ltd has a single shareholder, which is DEF Ltd. The director in question is also a director of DEF Ltd.

The equipment has been independently valued at £110,000. The net asset value of ABC Ltd is £2 million.

Which one of the following statements is correct regarding the need for shareholder approval for this transaction?

A) Shareholder approval by ordinary resolution will be required from the shareholders of ABC Ltd only.

B) Shareholder approval is not required because “father” is not a person connected to a director and therefore the acquisition does not fall within the legislation.

C) No shareholder approval is required, since value of the transaction is less than 10% of the company’s net asset value.

D) Shareholder approval by ordinary resolution will be required from the shareholders of DEF Ltd only.

E) Shareholder approval by ordinary resolution will be required from the shareholders of both ABC Ltd and DEF Ltd.

A

D) Shareholder approval by ordinary resolution will be required from the shareholders of DEF Ltd only

(Correct. This is a substantial property transaction with a director of the company and the holding company, therefore shareholder approval will be needed. Since ABC Ltd is a wholly owned subsidiary, no approval is required from its members)

85
Q

Company A has four shareholders, all of whom are also directors. The net asset value of Company A is £700,000.

Which one of the following transactions falls within the statutory provisions on substantial property transactions and will require shareholder approval from the shareholders of Company A by way of ordinary resolution?

A) The disposal of some equipment worth £75,000 by Company A to another company in which one of Company’s A’s directors has a 25% shareholding.

B) The purchase of some machinery worth £80,000 from the uncle of one of its directors.

C) The sale of some equipment worth £70,000 from Company A to one of its directors.

D) The purchase of some land worth £110,000 by Company A from the brother of one its directors.

A

A) The disposal of some equipment worth £75,000 by Company A to another company in which one of Company’s A’s directors has a 25% shareholding

(Correct. The value of the equipment exceeds 10% of the net asset value of the company and the disposal is to a connected person of one of the directors)

86
Q

Parent Plc owns 100% of the shares in Subsidiary Ltd. It is proposed that Subsidiary Ltd will lend the wife of one of Parent Plc’s directors £35,000 to start up a business.

Which of the following statements is correct in respect of the proposed loan?

A) Shareholder approval will be required from both Parent Plc and Subsidiary Ltd.

B) Shareholder approval will be required from Parent Plc but not Subsidiary Ltd.

C) The loan falls within the exceptions because it is under £50,000 and therefore no shareholder approval is needed.

D) Neither Parent Plc nor Subsidiary Ltd will need to obtain approval from their shareholders.

E) The loan does not fall within the relevant legislation because a wife of a director of a holding company does not fall within the definition of person connected to a director.

A

B) Shareholder approval will be required from Parent Plc but not Subsidiary Ltd.

87
Q

ABC Ltd has three directors and three shareholders (who are the three directors). It has no subsidiaries. ABC Ltd wishes to lend £50,000 to one of its directors. All of the directors and shareholders are in agreement that the loan should be made and that none of the directors will be breaching any of their general duties in providing the loan.

ABC Ltd seeks your advice as to the procedure to be followed in order that the loan may be provided as quickly as possible. Which of the following is the best advice?

A) Since the shareholders and directors are the same persons, no shareholder approval will be required for the loan. The board of ABC Ltd can pass a board resolution to approve the loan instead.

B) Shareholder approval by ordinary resolution will be required. ABC Ltd should prepare a memorandum setting out the terms of the proposed transaction and display this at the registered office. Since all the shareholders are in agreement, a general meeting may be held on short notice for the ordinary resolution to be passed.

C) Shareholder approval by ordinary resolution will be required. ABC Ltd should prepare a memorandum setting out the terms of the proposed transaction and submit this to the shareholders together with a written resolution to approve the loan. The ordinary resolution will be passed as soon as over 50% of the shareholders sign the written resolution.

D) Shareholder approval by ordinary resolution will be required. ABC Ltd should prepare a memorandum setting out the terms of the proposed transaction and display this at the registered office. The general meeting must be held on full notice for the ordinary resolution to be passed, due to the requirement for the memorandum to be displayed for 15 days ending with the date of the GM.

A

C) Shareholder approval by ordinary resolution will be required. ABC Ltd should prepare a memorandum setting out the terms of the proposed transaction and submit this to the shareholders together with a written resolution to approve the loan. The ordinary resolution will be passed as soon as over 50% of the shareholders sign the written resolution

(Correct. Shareholder approval will be required since this is a loan to one of the company’s directors and it does not fall within any of the exceptions. Since ABC Ltd wishes to provide the loan as quickly as possible, a written resolution should be used to avoid the 15-day notice requirement for the memorandum)

88
Q

ABC Ltd has two directors and four shareholders (who are the directors plus two other relatives of the directors). It has no subsidiaries.

For which one of the following transactions will ABC Ltd need shareholder approval by way of ordinary resolution?

A) ABC Ltd wishes to lend one of its directors £8,000 in order to pay for a training course.

B) ABC Ltd wishes to lend £20,000 to the husband of one of its directors.

C) ABC Ltd wishes to purchase a season travel ticket for one of its directors at a cost of £12,000. The director will repay the money in instalments on a salary sacrifice basis.

D) ABC Ltd wishes to pay £35,000 for renovations on the home of one of its directors, on the basis that the director will repay the money over a two-year period.

E) ABC Ltd wishes to provide a guarantee for a bank loan for £15,000 to one of its directors.

A

E) ABC Ltd wishes to provide a guarantee for a bank loan for £15,000 to one of its directors.

(Correct. This falls within s 197 therefore shareholder approval is required)

89
Q

A private limited company is planning to grant a service contract for a three-year term to one of its directors. The company was incorporated in 2010 and has adopted unamended model articles.

The company is keen for this to be approved by the shareholders as quickly as possible and has already been advised that this will need to be approved by way of an ordinary resolution.

All shareholders are understood to be generally available and responsive over the coming weeks.

Which of the following statements best describes the appropriate method the company should use?

A) The company should use the written resolution procedure as this should allow the process to be shortened significantly.

B) The company should call a general meeting under the usual notice requirements to ensure that all shareholders have an opportunity to participate in the voting.

C) The company should use the written resolution procedure to ensure that all shareholders have an opportunity to participate in the voting.

D) The company should use the short notice procedure in calling the general meeting as this should allow the process to be shortened significantly.

E) The company should use the short notice procedure in calling the general meeting to ensure that all shareholders have an opportunity to participate in the voting.

A

A) The company should use the written resolution procedure as this should allow the process to be shortened significantly

(Correct. The written resolution procedure is the only procedure that can be used to potentially bring down the time period significantly. The fact pattern indicates that all shareholders are expected to be available and responsive so there is no reason to suggest that a written resolution would take longer than a general meeting whether on short notice or not. The company will need to comply with the provisions of s 188(5)(a) Companies Act 2006 and with the general rules on the written resolution procedure – see s 288 Companies Act 2006 onwards. Use of the short notice procedure for the sake of time is incorrect given that, in the case of directors’ long term service contracts, s 188(5)(b) Companies Act 2006 requires a memorandum setting out the proposed contract to be made available to the members (i) at the company’s registered office for not less than 15 days ending with the date of the meeting and (ii) at the meeting itself. In reality, therefore, this would only shorten the ‘normal’ procedure by one day – i.e. 15 days instead of 14 clear days. Use of the written resolution procedure for the reasoning that it will allow all shareholders to have an opportunity to participate in the voting is incorrect as the scenario states that all shareholders are due to be available and responsive over the coming weeks)

90
Q

Your client is a manufacturing company (‘Company A’).

The managing director of Company A (‘the MD’) asks to speak to you about a proposed contract between Company A and a website design company (‘Company B’).

Company B is run by the MD’s close friend. The managing director has even invested in Company B himself and now owns 5% in Company B.

The MD wants to discuss this proposed contract at the upcoming board meeting of the 5 directors of Company A. The other directors of Company A are already aware that the MD has shares in Company B. Company A is a private limited company with unamended Model Articles.

Which of the following statements best summarises the advice you should give to the MD of Company A?

A) The MD should not declare their interest in the proposed contract with Company B as the other directors are already aware that the MD has an interest in the proposed transaction.

B) The MD should declare their interest in the proposed contract with Company B. If they do not, all 5 directors of Company A are at risk of breaching their directors’ duties contained within the Companies Act 2006.

C) The MD has a duty under s 175 of the Companies Act to avoid a conflict of interest. They are at risk of breaching that duty. They should therefore not propose that this contract is entered into at the board meeting of Company A.

D) The MD should declare their interest in the proposed contract with Company B at the board meeting of Company A. The MD will not be allowed to vote on the proposed contract at the board meeting.

E) The MD should sell their 5% shares in Company B to avoid a breach of their directors’ duties in the Companies Act 2006.

A

D) The MD should declare their interest in the proposed contract with Company B at the board meeting of Company A. The MD will not be allowed to vote on the proposed contract at the board meeting

(This is correct. This is the effect of Model Article 14 and s 177 of the Companies Act 2006)

91
Q

Director A is a director of Company B. Company B proposes to issue Director A with a service contract (the “Contract”) for a fixed term of three years (the “Term”) which contains the following provision at clause 11:

“The Company may not terminate the Contract before the expiry of the Term except for disciplinary reasons as set out in clause 15.”

What does Company B need to do before offering the Contract to Director A and what is the consequence of it failing to do so?

A) Company B must seek shareholder approval for the Contract by ordinary resolution. If it fails to do so, clause 11 of the Contract would be void and deemed replaced with a clause allowing Company B to terminate the Contract on reasonable notice.

B) Company B must seek shareholder approval for the Contract by special resolution. If it fails to do so, clause 11 of the Contract would be void and deemed replaced with a clause allowing Company B to terminate the Contract on reasonable notice.

C) Company B must seek shareholder approval for the Contract by ordinary resolution. If it fails to do so, clause 11 of the Contract would be void and deemed replaced with a clause allowing Company B to terminate the Contract on six months’ notice.

D) Company B must seek shareholder approval for the Contract by special resolution. If it fails to do so, the Contract will be void.

E) Company B must seek shareholder approval for the Contract by ordinary resolution. If it fails to do so, the Contract will be void.

A

A) Company B must seek shareholder approval for the Contract by ordinary resolution. If it fails to do so, clause 11 of the Contract would be void and deemed replaced with a clause allowing Company B to terminate the Contract on reasonable notice.

(Correct. This answer reflects the correct position. While the other answer option might sound plausible, they are each incorrect. Clause 11 of the Contract constitutes a “guaranteed term” of more than two years. As such, Company B cannot agree to such a provision unless it has been approved by the shareholders by ordinary resolution. If Company B enters into the Contract without the Term being approved by the shareholders as described, the contravening clause (clause 11) would be void to the extent of the contravention and the Contract would be deemed to contain a term entitling Company B to terminate it at any time by giving reasonable notice)

92
Q

A company (the “Company”) has three directors, a company secretary and four shareholders, each shareholder holding 25% of the Company’s share capital. The Company has unamended Model Articles of Association for Private Companies Limited by Shares. One of its directors would like a loan of £30,000 from the Company (the “Loan”) to fund some renovation work on personal property. The board of the Company would like to implement this Loan and do not anticipate the shareholders objecting to it. The Company does not use the written resolution procedure for such matters.

What resolutions (board and shareholder) are required for the Company to implement the Loan?

A) Shareholder resolution: Ordinary resolution to approve the loan.
Board resolutions: (1) Approve notice of general meeting, (2) convene general meeting, (3) instruct company secretary to make available to the shareholders a copy of the Loan agreement, (4) approve entry into the loan agreement, (5) authorise a director to sign the Loan agreement on behalf of the Company, (6) instruct company secretary to deal with post meeting matters.

B) Shareholder resolution: Special resolution to approve the loan.
Board resolutions: (1) Approve notice of general meeting, (2) convene general meeting, (3) instruct company secretary to make available to the shareholders a copy of the Loan agreement, (4) approve entry into the loan agreement, (5) authorise a director to sign the Loan agreement on behalf of the Company, (6) instruct company secretary to deal with post meeting matters.

C) Shareholder resolution: Ordinary resolution to approve the loan.
Board resolutions: (1) Approve notice of general meeting, (2) convene general meeting, (3) instruct company secretary to make available to the shareholders a memorandum setting out the nature, amount and purpose of the Loan, (4) approve entry into the loan agreement, (5) authorise a director to sign the Loan agreement on behalf of the Company, (6) instruct company secretary to deal with post meeting matters.

D) Shareholder resolution: Special resolution to approve the loan.
Board resolutions: (1) Approve notice of general meeting, (2) convene general meeting, (3) instruct company secretary to make available to the shareholders a memorandum setting out the nature, amount and purpose of the Loan, (4) approve entry into the loan agreement, (5) authorise a director to sign the Loan agreement on behalf of the Company, (6) instruct company secretary to deal with post meeting matters.

E) Shareholder resolution: Ordinary resolution to approve the loan.
Board resolutions: (1) Approve notice of general meeting, (2) convene general meeting, (3) instruct company secretary to make available to the shareholders a memorandum setting out the wording of the resolution to be passed, (4) approve entry into the loan agreement, (5) authorise a shareholder to sign the Loan agreement on behalf of the Company, (6) instruct company secretary to deal with post meeting matters.

A

C) Shareholder resolution: Ordinary resolution to approve the loan.
Board resolutions: (1) Approve notice of general meeting, (2) convene general meeting, (3) instruct company secretary to make available to the shareholders a memorandum setting out the nature, amount and purpose of the Loan, (4) approve entry into the loan agreement, (5) authorise a director to sign the Loan agreement on behalf of the Company, (6) instruct company secretary to deal with post meeting matters

(Correct. This answer reflects the correct position. While the other answer options might sound plausible, they are each incorrect. A company may not make a loan to its director unless the transaction has been approved by the shareholders passing an ordinary resolution. This is the only shareholder resolution required to implement the Loan. Since the Company does not use the written resolution procedure for such matters, a general meeting will need to be convened. A board meeting will therefore be required and board resolutions passed to convene the general meeting and approve the form of notice for the meeting. The ordinary resolution approving the Loan may not be passed unless a memorandum setting out the nature and amount of the Loan and the purpose for which it is required is made available for inspection by the shareholders for at least 15 days ending with the date of the general meeting at which it is voted on and at the general meeting itself. Once the ordinary resolution approving the Loan is passed by the shareholders, a second board meeting is required to implement the Loan. This will entail passing board resolutions to approve entering into the loan agreement and appointing a director to sign the document on behalf of the Company. A board resolution requiring the company secretary to deal with post meeting matters is also required)

93
Q

A private limited company, which operates a publishing business, has no subsidiaries and is owned by individual shareholders. One of its directors wishes to undertake some home improvements and has obtained a quotation of £8,000 from a firm of builders. The director has asked the other board members if the company would pay this sum to the builders up-front, on condition that the director repay the company in monthly instalments over the coming year. The board is happy to approve such an arrangement and has asked you whether or not it also requires shareholder approval.

Which of the following comprises the best advice to the company’s board?

A) An ordinary resolution is necessary because the transaction is a credit transaction.

B) No shareholder resolution is necessary because the Companies Act 2006 does not require approval of this transaction.

C) No shareholder approval is necessary because the value of the transaction is below £10,000.

D) An ordinary resolution is necessary because the transaction is a quasi-loan.

E) No shareholder resolution is necessary because the company is not a wholly-owned subsidiary of any other company.

A

B) No shareholder resolution is necessary because the Companies Act 2006 does not require approval of this transaction

(Correct. No shareholder resolution is necessary. The transaction described is a quasi-loan, as defined in s 199 Companies Act 2006. The requirement in s 198 (2) for shareholders to approve a quasi-loan to a director of a company applies only if such company is either (i) a public company or (ii) a private company associated with a public company. The company in this case is a private limited company. Because it is owned by individuals and has no subsidiaries, we can conclude that it is not associated with a public company (s 256))

94
Q

Which of the following is the correct threshold shareholding for a shareholder to give them the right to require the directors to call a general meeting?

A) 25%

B) 20%

C) 5%

D) 1%

E) 10%

A

C) 5%

(Correct. Shareholders together holding not less than 5% of the paid-up voting share capital of the company can require a general meeting, by serving a request on the company.)

95
Q

You are advising a company on its incorporation. There are five directors who are all equal shareholders.

They want to ensure that none of them can be removed as a director without unanimous consent.

Which of the following statements represents the best advice to the shareholder/directors?

A) There is nothing that they can do to prevent removal of a director by a majority of shareholders under s 168 CA 2006.

B) The shareholders should enter a shareholders’ agreement requiring unanimity to remove a director.

C) The shareholders should amend the articles to require unanimity to remove a director.

D) A shareholders’ agreement would not necessarily be helpful as it could be amended with support of a simple majority of the shareholders.

E) The company should make sure it is party to the whole shareholders’ agreement to ensure it is legally binding on both the company and the members.

A

B) The shareholders should enter a shareholders’ agreement requiring unanimity to remove a director.

96
Q

An accountant has purchased some shares in a company on the basis that their accountancy firm would be instructed to prepare the annual accounts for the company. To be cautious the accountant required that a formal statement was inserted into the articles of the company to this effect. No other agreement was entered into on purchasing the shares or otherwise.

Which of the following statements would represent the correct advice to the accountant?

A) They would have a claim under s 33 CA 2006 because the articles represent a contract between the company and the shareholders.

B) They would not have a claim under the articles as rights which do not relate to their membership are not enforceable under s 33 CA 2006.

C) Being appointed in a professional capacity to advise the board is likely to constitute a membership right.

D) The articles are only advisory so are not legally binding as between the accountant and the company.

E) The court might imply terms into the articles in favour of the accountant if they can show such an implied term would create business efficacy.

A

B) They would not have a claim under the articles as rights which do not relate to their membership are not enforceable under s 33 CA 2006.

97
Q

You act for XYZ Ltd, a company with unamended model articles and five shareholders, all of whom are also directors. All of the shareholders hold 20% of the shares.

Three of the shareholders are unhappy with one of the directors and wish to remove them as a director. The other shareholder is not in favour of the resolution.

On the incorporation of the company all five of the shareholders signed a shareholders agreement stating that all shareholders must vote unanimously to remove a director.

Which of the following statements is correct about the resolution to remove the director?

A) The two opposing shareholders can block the resolution to remove the director.

B) If the resolution is passed to remove the director, they will likely have a claim for breach of the articles against the shareholders that voted for the resolution.

C) It will not be possible to pass the resolution to remove the director under s 168 CA 2006 because of the clause in the shareholders agreement requiring unanimity.

D) It will be necessary to check the articles to check there are no Bushell v Faith Clauses.

E) If the resolution is passed to remove the director, they will likely have a claim for breach of the shareholders’ agreement against the shareholders that voted for the resolution.

A

E) If the resolution is passed to remove the director, they will likely have a claim for breach of the shareholders’ agreement against the shareholders that voted for the resolution.

98
Q

A company has received a notice from its shareholders which purports to be served under s 168 CA 2006 in order to move a resolution to remove a director.

The shareholders signing the s 168 notice holds 4% of the shares of the company. The company does not think that any of the other shareholders support this notice as the shareholder has a history of serving vexatious s 168 notices.

Which of the following represents the best advice to the board?

A) If the board does not put this on the agenda of the GM, it will not need to tell the affected director.

B) If the board chooses not to out this on the agenda, the shareholder can serve a s 303 CA 2006 notice requiring the directors to call a GM to consider the issue.

C) The board has to put this on the agenda of the next GM.

D) The board does not need to put this on the agenda of a GM to be considered.

E) If the board chooses to put this on the agenda of a GM they will have 21 days to call the GM, and a further 28 days to hold it.

A

D) The board does not need to put this on the agenda of a GM to be considered.

99
Q

A company has received a notice from its shareholders which purports to be served under s 168 and s 303 of the Companies Act 2006 in order to move a resolution to remove a director.

The shareholders signing the notice together hold 10% of the shares of the company. If the company receives the notice today (Day 1) which of the statements is correct about the latest day on which the company can hold the General Meeting, assuming the directors want to co-operate with the Companies Act 2006 but also use all the time available to them?

A) Day 30

B) Day 50

C) Day 51

D) Day 16

E) Day 49

A

B) Day 50

100
Q

Which of the following is the correct definition of a derivative claim?

A) A claim brought by a member seeking for the company to be wound up.

B) A claim brought by a member in respect of a wrong done to the member personally.

C) A claim brought by a member seeking for the company to buy their shares.

D) A claim brought by a member in respect of a cause of action vested in the company seeking relief on behalf of the company.

A

D) A claim brought by a member in respect of a cause of action vested in the company seeking relief on behalf of the company.

101
Q

A company secretary has received a letter of complaint from one of its shareholders. The shareholder, who owns 5% of the shares, is unhappy that the company has not dealt with a breach of duty by one of the directors who has involved his wife in the breach. The director’s wife is not a director.

Which of the following statements represents the correct advice about a claim under s 260 CA 2006 by the shareholder?

A) The claim will be brought by the shareholder in their personal capacity.

B) The shareholder must bring the claim against all of the directors who will be jointly and severally liable under s 260 CA 2006.

C) The claim will be brought against the company as a body corporate.

D) The shareholder can bring a claim against the defaulting director and his wife.

E) It will not be possible to include the wife in the action as she is not a director.

A

D) The shareholder can bring a claim against the defaulting director and his wife.

102
Q

A company secretary has received a letter from one of its former shareholders threatening to bring a derivative claim under s 260 CA 2006. The shareholder sold all their shares 12 months ago to the company.

The claim relates to a one-off breach of duty by two of the directors that happened 18 months ago

The company issued shares this week to a new shareholder.

Which of the following statements represents the correct advice about a potential claim under s 260 CA 2006 by the shareholder?

A) The complaining shareholder will be able to bring a claim if they can prove the default happened at a time when they held shares in the company.

B) The complaining shareholder will not be able to bring a derivative claim because they are no longer a member.

C) The new shareholder will not be able to bring a derivative action because they were not a shareholder at the time of the alleged default.

D) The new shareholder can bring a claim if they join a shareholder who was a member at the time of the default into the action.

E) It is not possible for anybody to bring a claim under s 260 CA 2006 as the claim has not been brought as soon as is practicable.

A

B) The complaining shareholder will not be able to bring a derivative claim because they are no longer a member.

103
Q

You act for a shareholder in a company which was incorporated five years ago with unamended model articles. There were originally three equal shareholders who, until recently, were the only three directors. There is a shareholders’ agreement but it does not contain any provisions dealing with the voting on any removal of a director. Over the years more shareholders have invested and so your client’s shareholding is now 20%.

Two weeks ago, your client was removed from the board by an ordinary resolution. She no longer wants to be part of the company. Which of the following claims is the most appropriate claim she could bring?

A) Derivative action under s 260 CA 2006.

B) Breach of the shareholders agreement for removing her as director.

C) Unfair prejudice under s 994 CA 2006.

D) Breach of membership rights under s 33 CA 2006.

E) No claims are open to her as being removed as a director and being a shareholder are separate roles.

A

C) Unfair prejudice under s 994 CA 2006.

104
Q

Which of the following is the most common remedy that the court will order in a claim for unfair prejudice?

A) An order that the company or the other shareholders purchase the petitioner’s shares.

B) An order that the company pays damages to the petitioner to reflect the diminution in value of their shareholding.

C) An order that the company be wound up.

D) An order regulating the conduct of the company in the future.

A

A) An order that the company or the other shareholders purchase the petitioner’s shares.

105
Q

Which of the following correctly summarises the requirements for a successful claim for unfair prejudice?

A) The petitioner must show that the company’s affairs are being or have been conducted in a manner which is unfairly prejudicial to the interests of its members generally or of some part of its members.

B) The petitioner must show that the company is insolvent.

C) The petitioner must show that the company’s affairs are being or have been conducted in a manner which is unfairly prejudicial to all of the members.

D) The petitioner must show that the directors have acted outside of the articles.

A

A) The petitioner must show that the company’s affairs are being or have been conducted in a manner which is unfairly prejudicial to the interests of its members generally or of some part of its members.

106
Q

You are advising an executive director of a private company limited by shares (with unamended Model Articles) in respect of the upcoming general meeting agenda item of his removal from the board of directors by the shareholders.

Your client is not only a director, but he is also a minority shareholder in the company with 10% of the company’s issued share capital. All of the shareholders in the company previously signed a shareholders’ agreement which included the following clause: ‘The shareholders shall, for as long as they hold shares in the capital of the company, procure that the company shall not without the prior written consent of all shareholders remove any director.’

In light of the above, which of the following statements represents the advice you would give?

A) Your client can be removed as a director of the company by the shareholders passing an ordinary resolution. As your client has an interest in his own removal, he will not be able to vote against his removal in the general meeting. If removed, your client could bring a claim for breach of contract against the other shareholders who voted in favour of his removal.

B) Your client can not be removed as a director of the company as he is an employee of the company in his role as an executive director. As an employee of the company his position as a director is entrenched. If he was removed as a director, this would automatically also end his employment and breach his employment contract.

C) Your client can be removed as a director of the company by the shareholders passing an ordinary resolution. However, if removed, your client could bring a claim for breach of contract against the other shareholders who voted in favour of his removal.

D) Your client can be removed as director of the company by the shareholders passing an ordinary resolution. However, if removed, your client could bring a claim for breach of contract against the company.

E) Your client can not be removed as a director. The shareholders’ agreement requires unanimous consent for the removal of a director and your client will not vote in favour of his own removal.

A

C) Your client can be removed as a director of the company by the shareholders passing an ordinary resolution. However, if removed, your client could bring a claim for breach of contract against the other shareholders who voted in favour of his removal.

(Correct. The shareholder’s vote would still be effective under s 168(1) CA 2006, but your client would be able to bring an action for breach of contract (i.e. the shareholders’ agreement). Remember that this agreement was a private agreement amongst the shareholders and does not bind the company.)

107
Q

You act on behalf of an individual shareholder with a minority shareholding (10% of the company’s shareholding). The client has recently been unwillingly removed as a director of the company and dismissed as an employee of the company. Your client has been a shareholder in the company for 3 years (since the company was incorporated). The other 3 shareholders in the company are also the only directors in the company. The company is a small private company limited by shares (with unamended Model Articles).

Which of the following actions is both available to your client and most likely to be successful based on the information available?

A) Your client should bring a petition for the just and equitable winding up of the company.

B) Your client should pursue a derivative action on behalf of the company. The remaining directors of the company are likely to have breached their directors’ duties by deciding to dismiss your client as an employee.

C) Your client should pursue an unfair prejudice claim. The company is likely to qualify as a quasi-partnership. If successful, it is likely that the court would order the purchase of your client’s shares by the other shareholders or by the company.

D) Your client should bring a claim under s 33 Companies Act 2006 for breach of their membership rights. The most likely remedy is damages.

E) Unfortunately there is no action that your client can take in these circumstances.

A

C) Your client should pursue an unfair prejudice claim. The company is likely to qualify as a quasi-partnership. If successful, it is likely that the court would order the purchase of your client’s shares by the other shareholders or by the company

(Correct. This will provide your client with a mechanism to recover their investment in the company without the difficulty of having to find an external investor to buy their shares. Based on the information you have been provided this action is most likely to be successful)

108
Q

The shareholders of a company are dissatisfied with the performance of one of the directors of the company and wish for the director to be removed. The board of directors as a whole is loyal to the underperforming director and is unlikely to take any action to remove them. The company has unamended Model Articles.

What notices and/or requests should the shareholders immediately send to the board in order to ensure that the director is removed as quickly as possible?

A) A request for the directors to circulate a written resolution to the board.

B) A special notice of the proposed resolution for removal and a request requiring the directors to call a general meeting.

C) A special notice of the proposed resolution for removal and a request for the directors to circulate a written resolution.

D) A request requiring the directors to call a general meeting.

E) A special notice of the proposed resolution for removal.

A

B) A special notice of the proposed resolution for removal and a request requiring the directors to call a general meeting.

(Correct. Shareholders can remove a director by passing an Ordinary Resolution under s 168(1) CA 2006. Section 168(2) requires the shareholders to serve special notice of the proposed resolution. The directors are not obliged to place the proposed resolution on the agenda at a meeting of the shareholders (Pedley v Inland Waterways Association Ltd) and on the facts they are unlikely to. The shareholders can call a general meeting themselves in accordance with s 303 CA 2006. In order for unhappy shareholders to ensure the resolution to remove a director is heard as soon as possible, they will submit a s 303 CA 2006 request requiring the directors to call a general meeting at the same time as sending their s 312 CA 2006 special notice to the board. The written resolution procedure cannot be used to remove a director (s 288(2)(a) CA 2006))

109
Q

A is a company director. Shareholders holding 7% (the ‘Shareholders’) of the share capital of the company have served notice on the company board of directors (the ‘Board’) of intention to remove A as a director. The next general meeting is due to be held in exactly one calendar months’ time. The company has articles in the form of unamended model articles.

Which of the statements below provides the best advice to the Board concerning the resolution to remove A as a director (the ‘Resolution’)?

A) The Shareholders do not represent sufficient of the voting rights of the company to have the right to call a general meeting to move the Resolution.

B) The Board has 28 days in which to decide whether to put the Resolution on the agenda of the next general meeting.

C) The Board should put the Resolution on the agenda for the upcoming general meeting, since if the Board does not do so, then the Shareholders can call a general meeting to move the Resolution.

D) The Board can refuse to put the Resolution on the agenda of the next general meeting.

E) The Shareholders have not given sufficient notice to move the Resolution for the upcoming general meeting, therefore the Board do not need to put the Resolution on the agenda for this general meeting.

A

C) The Board should put the Resolution on the agenda for the upcoming general meeting, since if the Board does not do so, then the Shareholders can call a general meeting to move the Resolution

(Correct. The special notice period can be observed. Although technically the Board may refuse to place the Resolution on the agenda of the next general meeting (Pedley v Inland Waterways), it would be unwise to do so as the Shareholders meet the 5% voting threshold to call the general meeting if the Board fails to do so within 21 days of notice of intention to remove a director)

110
Q

You are acting on behalf of a private company limited by shares (currently with unamended Model Articles). The 4 individual shareholders of the company are also the only 4 directors of the company. The board of directors wish to consider ways to prevent any director being removed against their will. The board has consequently asked for ways to prevent the removal of directors taking place without the prior written consent of all of them.

In light of the above, where, if anywhere, is the most appropriate place for a provision dealing with this issue to be set out?

A) Nowhere is appropriate for this provision as the provision is contrary to the CA 2006 and so is legally unenforceable.

B) The company’s articles of association.

C) A shareholders’ agreement or the company’s articles of association.

D) A shareholders’ agreement.

E) The contract of employment of each director.

A

D) A shareholders’ agreement

(Correct. Remember that any provision in the articles which requires the company to restrict its statutory powers in any way will be void. Shareholders on the other hand may deal with how they should exercise their voting rights however they so agree. Any such provision should be set out in a shareholders’ agreement and not the articles so that such a provision is a personal obligation between the shareholders only and not a restriction on the company)

111
Q

A company is looking to issue some new shares to an outside investor. The shares will have the following rights attached to them:

· The holder shall receive a preference as to dividend of 5% of the nominal value per annum.

· On winding up, the holder shall receive as a preference the nominal value of the share, if there is sufficient capital to pay.

· The right to the preference dividend is cumulative.

· The holder shall have no right to any other dividend or any return on capital.

· The shares carry no rights to vote.

Which of the following statements best describes the shares?

A) 5% cumulative preference shares.

B) 5% ordinary shares.

C) 5% participating cumulative preference shares.

D) 5% preference redeemable shares.

E) 5% non-cumulative preference shares.

A

A) 5% cumulative preference shares.

112
Q

A company is looking to issue some new shares to an outside investor. The shares will have the following rights attached to them:

· The holder shall receive a preference as to dividend of 4% of the nominal value per annum.

· On winding up, the holder shall receive as a preference the nominal value of the share, if there is sufficient capital to pay.

· The right to the preference dividend is not cumulative.

· The holder shall have the right participate in the general dividend and return on capital.

· The shares carry no rights to vote.

Which of the following statements best describes the shares?

A) 4% non-participating preference shares.

B) 4% redeemable shares.

C) 4% participating non-cumulative preference shares.

D) 4% participating cumulative preference shares.

E) 4% non-participating non-cumulative preference shares.

A

C) 4% participating non-cumulative preference shares.

113
Q

When does a shareholder acquire full legal title to new shares that the company has issued to the shareholder?

A) When they acquire the unconditional right to be included in the company’s register of members in respect of those shares.

B) When their name is entered into the company’s register of members.

C) When they receive their share certificate.

D) On payment for the shares.

E) When the company notifies Companies House that the shares have been issued.

A

B) When their name is entered into the company’s register of members.

114
Q

When does a shareholder acquire full legal title to shares that are transferred to them from an existing shareholder?

A) When the stock transfer form is executed.

B) When their name is entered into the company’s register of members.

C) When they receive their share certificate.

D) On payment for the shares.

E) When they have paid the stamp duty payable or confirmed that the transfer is exempt from stamp duty.

A

B) When their name is entered into the company’s register of members

Correct. See section 112(2) CA 2006.

115
Q

In which one of the following circumstances is a stock transfer form required to transfer the shares to the new shareholder?

A) When a shareholder dies and their shares are passed to their personal representatives.

B) When a shareholder is made bankrupt and their shares are passed to their trustee in bankruptcy.

C) When a company issues new shares.

D) When an existing shareholder of the company gifts their shares to a family member during their life.

A

D) When an existing shareholder of the company gifts their shares to a family member during their life.

Correct. This is a transfer of shares and therefore a stock transfer form is required.

116
Q

A company has two shareholders, A and B. A wishes to transfer all of their shares to B. B will pay market value for the shares of £15,000.

Which of the following statements is correct about the transfer of shares proposed?

A) No stamp duty is payable as this share transfer is not caught by the legislation.

B) Legal title will pass to B on execution of the stock transfer form.

C) Stamp duty of 0.5% of the value of the transfer will be payable be B.

D) The company will need to be party to the transfer agreement.

E) Both A and B will need to sign the stock transfer form.

A

C) Stamp duty of 0.5% of the value of the transfer will be payable be B.

117
Q

A private limited company, which was incorporated in September 2018, currently has an issued share capital of £1000 made up of 1000 £1 ordinary shares. The shares are held equally by five shareholders.

The company now wishes to issue 150 £10 2% preference shares to an outside investor. The preference shares entitle the shareholder to receive only a fixed dividend of 2% per annum, with no right to share in any surplus profits. The shareholder is only entitled to the return of the nominal value of the shares on winding up of the company.

The company has Model Articles with one amendment; this new preference share has already been included in the Articles.

Which of the following statements represents the best advice to the company about the resolutions required for the proposed share allotment?

A) The company will need a special resolution to amend the articles.

B) The company will need a special resolution from the shareholders to give the directors authority to allot the shares.

C) The company will need a special resolution from the shareholders to disapply pre-emption rights.

D) The company will need an ordinary resolution from the shareholders to give the directors authority to allot the shares.

E) The company will not need any shareholder resolutions, only a board resolution to allot.

A

D) The company will need an ordinary resolution from the shareholders to give the directors authority to allot the shares.

118
Q

A company is proposing to issue shares to a new shareholder. The shares will give the shareholder the right to a fixed dividend of 2% of the nominal value of the shares in preference to the other shareholders with no other right to share in any surplus profits. In respect of capital return on a winding up the shareholder will receive firstly, as a preference, the nominal value of the share and then the shareholder will rank pari pasu with the ordinary shareholders if there is any surplus capital.

Which of the following statements best describes the shares and any resolutions required as a result of them?

A) The company will need an ordinary resolution to disapply pre-emption rights because the shares are an equity security.

B) The company will need unanimity from its shareholders to agree to disapply the pre-emption rights because the shares are equity securities.

C) The company will not need to disapply pre-emption rights as the shares are not equity securities.

D) The company will not need to disapply pre-emption rights because these shares will not dilute the current shareholdings.

E) The company will need a special resolution to disapply pre-emption rights because the shares are equity securities.

A

E) The company will need a special resolution to disapply pre-emption rights because the shares are equity securities.

119
Q

You act for a company incorporated under the CA 1985. The company has an issued share capital of 1000 £1 ordinary shares and an authorised share capital of £2000. The company wishes to issue a further 1000 £1 ordinary shares to an outside investor.

The company has not passed any shareholder resolutions which would be relevant to this question.

Which of the following statements represents the correct shareholder resolutions that the company would need to issue these shares?

A) An ordinary resolution to remove the cap, an ordinary resolution to give the directors authority to allot the shares and a special resolution to disapply pre-emption rights.

B) An ordinary resolution to give the directors authority to allot the shares and a special resolution to disapply pre-emption rights.

C) An ordinary resolution to remove the cap and an ordinary resolution to give the directors authority to allot the shares.

D) An ordinary resolution to remove the cap and a special resolution to disapply pre-emption rights.

E) A special resolution to disapply pre-emption rights only.

A

B) An ordinary resolution to give the directors authority to allot the shares and a special resolution to disapply pre-emption rights.

120
Q

A company is looking to purchase the entire issued share capital of a private limited company (the ‘Target’). The Target has two wholly owned subsidiaries: one of which is a PLC and the other is a private limited company. The Buyer is proposing to fund the acquisition wholly with a loan from the bank. In return for the loan, the bank requires security over the assets of the Target, both of the Target’s subsidiaries and the Buyer.

Which of the following statements is correct in respect of prohibited financial assistance?

A) Only the security proposed by the Plc subsidiary is caught by the prohibited financial assistance regime.

B) The security offered by the target itself will be caught by the prohibited financial assistance regime.

C) None of the securities will be caught because the target is private.

D) The security offered by the Target and both of the subsidiaries are caught by the prohibited financial assistance regime.

E) The security offered by all four companies is caught by the prohibited financial assistance regime.

A

A) Only the security proposed by the Plc subsidiary is caught by the prohibited financial assistance regime.

121
Q

A company is looking to purchase the entire issued share capital of a public limited company (the ‘Target’). The Target has two wholly owned subsidiaries: one of which is a plc and the other is a private limited company. The Buyer is proposing to fund the acquisition wholly with a loan from the bank. In return for the loan, the bank requires security over the assets of the Target, both of the Target’s subsidiaries and the Buyer.

Which of the following statements is correct in respect of prohibited financial assistance?

A) The security offered by the Target and both subsidiaries will be caught by the prohibited financial assistance regime.

B) Only the security offered by the PLC subsidiary will be caught by the prohibited financial assistance regime.

C) The security offered by the Target and the PLC subsidiary will be prohibited financial assistance but not the other companies.

D) Only the security offered by the target will be caught by the prohibited financial assistance regime.

E) All of the security offered by all four companies will be caught by the prohibited financial assistance regime

A

A) The security offered by the Target and both subsidiaries will be caught by the prohibited financial assistance regime.

122
Q

A company (the ‘Buyer’) is looking to purchase the entire issued share capital of a private limited company (the ‘Target’). The Target is a wholly owned subsidiary of a PLC. The Buyer is proposing to fund the acquisition partly with a loan from the bank. In return for the loan, the bank requires security over the assets of the Target, the parent company and the Buyer.

Which of the following statements is correct in respect of prohibited financial assistance?

A) All of the security options fall within the prohibited financial assistance regime.

B) None of the security options fall within the prohibited financial assistance regime.

C) Both the security of the Target and its parent are caught.

D) Only the security from the parent of the Target company is caught by the prohibited financial assistance regime.

E) Only the security from the Target company is caught by the prohibited financial assistance regime.

A

B) None of the security options fall within the prohibited financial assistance regime.

123
Q

A company has a class of redeemable shares which it is looking to redeem. Where will the terms of redemption of those shares be set out?

A) The terms on which redeemable shares may be redeemed are determined at the time that the shares are issued and are set out in the company’s articles.

B) The terms on which redeemable shares may be redeemed are determined at the time that the shares are issued and set out in a shareholders’ agreement.

C) The terms on which redeemable shares may be redeemed are determined at the time that the shares are issued. They are set out in a contract signed by the company and the shareholders.

D) The company must draw up a contract setting out the terms on which it will redeem the shares. This must be approved by an ordinary resolution of the shareholders.

A

A) The terms on which redeemable shares may be redeemed are determined at the time that the shares are issued and are set out in the company’s articles.

124
Q

Which of the following best summarises the principle of maintenance of capital?

A) The share capital of a company is represented in the bottom half of the balance sheet. These sums may never be released to shareholders.

B) The share capital of a company is seen as a permanent fund available to its creditors. Companies are never permitted to return capital to shareholders.

C) The share capital of a company is seen as a permanent fund available to its creditors. Companies are not permitted to return capital to shareholders other than in limited circumstances.

D) The share capital of a company is seen as a permanent fund available to its creditors. Buybacks are only allowed for private limited companies and then only in limited circumstances.

A

C) The share capital of a company is seen as a permanent fund available to its creditors. Companies are not permitted to return capital to shareholders other than in limited circumstances.

125
Q

A private limited company with unamended model articles and four shareholders with equal shareholdings is seeking to buyback its shares from one of its shareholders. The company does not have sufficient distributable profits to fund the buyback and is also not able to fund the buyback out of the proceeds of a fresh issue of shares. The company therefore wishes to use capital to fund the buyback. What shareholder resolution(s) will be necessary in order to approve this buyback?

A) An ordinary resolution to approve the terms of the contract for the buyback only.

B) An ordinary resolution to approve the terms of the contract for the buyback and an ordinary resolution to approve the payment out of capital.

C) An ordinary resolution to approve the terms of the contract for the buyback and a special resolution to approve the payment out of capital.

D) A special resolution to approve the terms of the contract for the buyback and a special resolution to approve the payment out of capital.

E) A special resolution to approve the payment out of capital only.

A

C) An ordinary resolution to approve the terms of the contract for the buyback and a special resolution to approve the payment out of capital.

126
Q

A taxpayer has the following receipts in the current tax year:

a) £10,000 from the sale of an oil painting inherited from their mother 3 years’ ago;

b) £5,000 from the sale of shares in XYZ plc;

c) £7,500 as an annual dividend from the shares they own in ABC Limited.

Which of the following statements represents the correct statement about whether the above amount to income or capital receipts?

A) The receipt from the sale of the oil painting is a capital receipt and the receipts from the sale of the shares in XYZ Plc and dividend are income receipts.

B) The receipts from the sale of the oil painting and shares in XYZ Plc are income receipts and the dividend is a capital receipt.

C) All of the receipts are income receipts.

D) All of the receipts are capital receipts.

E) The receipts from the sale of the oil painting and shares in XYZ Plc are capital receipts and the dividend is an income receipt.

A

E) The receipts from the sale of the oil painting and shares in XYZ Plc are capital receipts and the dividend is an income receipt.

127
Q

A taxpayer who runs a printing business as a sole trader has the following expenditure in the current tax year:

a) £15,978 for heating and lighting of business premises;

b) £6,500 for an advertising campaign;

c) £17,000 on a new printing press machine.

Which of the following statements correctly identifies whether these expenses represent income or capital expenditure?

A) All of the expenditure is income expenditure.

B) The expenditure on heating and lighting is income expenditure. The expenditure on the advertising campaign and the new printing press machine is capital expenditure.

C) The expenditure on heating and lighting and the advertising campaign are income expenditure. The expenditure on the new printing press machine is capital expenditure.

D) All of the expenditure is capital expenditure.

E) The expenditure on heating and lighting and the advertising campaign are capital expenditure. The expenditure on the new printing press machine is income expenditure.

A

C) The expenditure on heating and lighting and the advertising campaign are income expenditure. The expenditure on the new printing press machine is capital expenditure.

128
Q

A taxpayer runs an antiques shop and owns a portfolio of shares. Last week:

a) The shop sold a painting and a grandfather clock;

b) The taxpayer sold 15% of their share portfolio.

Which of the following statements represents the correct statement about whether the above amount to income or capital receipts?

A) All of the receipts will be capital receipts.

B) The sums received from the sale of the painting and the grandfather clock will be capital receipts and the sum received from the sale of the share portfolio will be an income receipt.

C) The sum received from the sale of the painting will be capital a receipt and the sums received from the sale of the share portfolio and the grandfather clock will be income receipts.

D) The sums received from the sale of the painting and the grandfather clock will be income receipts and the sum received from the sale of the share portfolio will be a capital receipt.

E) All of the receipts will be income receipts.

A

D) The sums received from the sale of the painting and the grandfather clock will be income receipts and the sum received from the sale of the share portfolio will be a capital receipt.

129
Q

A man is to receive a cash dividend of £35,000 in the current tax year. He has other taxable income of £26,225 (all of which is non-savings income).

Which one of the following options is the correct amount of tax payable to HMRC by the man on the dividend he receives?

(round down your figures to the nearest pound at each stage of your calculation)

£11,475

£11,812

£2,975

£8,943

£8,855

A

£8,855

Correct. The correct calculation is:
Dividend £35,000 Other non-savings income £26,225 Available Basic Rate (£37,700 - £26,225) £11,475 Dividend will be taxed as follows: £1,000 @ 0% dividend nil rate £0 Remaining Basic Rate (£11,475 - £1,000) £10,475 @ 8.75% £916 Higher rate for remaining(£35,000 - £1,000 - 11,475) £23,525 @ 33.75% £7,939
Total tax payable on dividend £8,855

130
Q

A taxpayer is a partner in a small catering business. The following information relates to their income and other related tax affairs for the current financial year.

· Trading profits for tax purposes £73,000

· Savings related income £2,250

· Receipt of £5,000 from the sale of a painting

· Dividend on shares in ICR Plc £1,000

· Contributions made by the taxpayer into a personal pension scheme £5,000

· Interest on a loan the taxpayer took out to inject further capital into the business £3,500 per annum

What is the taxpayer’s net income?

£76,250

£67,750

£66,750

£71,250

£72,750

A

£67,750

Correct. Add up all of the sources of income to work out Total Income - trading profit of £73,000, plus savings income of £2,250 and dividend income of £1,000 = £76,250.
To work out the Net Income figure, you subtract available tax reliefs, which in this case are the pension contributions of £5,000 and the interest on the loan to invest in the business of £3,500 = £8,500.
£76,250 - £8,500 = £67,750 (Net Income).
The receipt of £5,000 for the sale of the painting is a one off transaction and so is a capital receipt, not income.

131
Q

A taxpayer has received £5,000 in respect of interest on a savings account they have with an online bank. They already have taxable income (which is all non-savings income) of £35,000. The current basic rate tax threshold is £37,700.

Which of the following statements represents the correct amount of tax the taxpayer should pay on the interest?

£1,460

£1,000

£900

£1,800

£1,360

A

£1,360

Correct.
The correct calculation is:
Interest 5,000 Other non-savings income 35,000 Available Basic Rate (37,700 – 35,000) 2,700 Interest will be taxed as follows: £500 @ 0% PSA nil rate £0 Remaining Basic Rate (2,700-500) 2200 @ 20% £440 Higher rate for remaining 2,300 @ 40% £920
Total tax payable on savings£1360

132
Q

A taxpayer is a shareholder of XYZ Ltd, a trading company. XYZ Ltd was incorporated in 2004 and the taxpayer has held her shares in XYZ Ltd since incorporation and has worked for the company since that time. The taxpayer owns 1500 of the 2000 shares in issue. She would like to sell 200 of her shares to her friend for £225,000.

All shares in XYZ Ltd are ordinary voting shares and all were issued in 2004 at £1.50 per share. All shares carry an entitlement to profits and assets available for distribution to shareholders on a winding up. The taxpayer did not incur any incidental acquisition costs. The legal fees for the taxpayer to transfer title in the shares to her friend have been estimated at £900.

What is the capital gains tax that would be payable onthis disposal? Ignore any other gains/losses the taxpayer may have made in the current, or any previous, tax year and consider any applicable relief.

We cannot say as we do not know her income.

£21,870

£22, 380

£21,780

£21,810

A

£21,780

Correct. The correct calculation is:
Consideration £225,000
Less costs of disposal (£900)
Net sale proceeds: £224,100
Less base cost 200 shares @ £1.50 (£300)
Total chargeable gain £223,800
Less AE (£6,000)
Taxable chargeable gain: £217,800
Taxed at 10% as TP would be entitled to Business Asset Disposal Relief = £21,780

133
Q

A taxpayer is disposing of a tea shop business on which they have made a significant gain since purchasing the shop in 1994. They are working out which expenditure they will be able to deduct from the consideration to reduce their tax liability. Which one of the following pieces of expenditure is NOT an allowable expenditure for capital gains tax purposes?

A) Replacement of roof tiles to the shop following a storm in 2006.

B) The cost of an extension to the café area of the tea shop to allow more customers in.

C) The original cost of the business in 1994.

D) Solicitor’s fees on dealing with the sale.

E) The surveyor’s fees incurred when they purchased the shop in 1994.

A

A) Replacement of roof tiles to the shop following a storm in 2006.

134
Q

A taxpayer has a taxable chargeable gain on the sale of some shares which the taxpayer had as an investment in ABC Ltd, a private limited company which is currently trading. The taxpayer bought the shares nine years ago on the incorporation of the company. The taxpayer is not an employee or officer of ABC Ltd.

The taxpayer held 10% of the shares and sold them all in this tax year.

Which of the following statements represents the correct advice concerning the taxation of the chargeable gain?

A) The taxable chargeable gain will all be taxed at 20%.

B) The taxpayer will benefit from Business Asset Disposal relief because all of the criteria are met.

C) The taxpayer will not benefit from Business Asset Disposal relief because not all of the criteria are met.

D) The taxpayer will not benefit from Business Asset Disposal relief because the sale of shares is not something to which the relief applies.

E) We do not have enough information to establish if Business Asset Disposal Relief is available.

A

C) The taxpayer will not benefit from Business Asset Disposal relief because not all of the criteria are met.

135
Q

Business Property Relief is available on which type of taxation?

Capital Gains Tax

VAT

Corporation Tax

Inheritance Tax

Income Tax

A

Inheritance Tax

136
Q

Which type of chargeable transfers is Business Property Relief applicable to?

Only transfers on death

PETs only

LCTs only

PETs, LCTs and death

PETs and LCTs but not death

A

PETs, LCTs and death

137
Q

Which of the following is NOT a qualifying business asset for Business Property Relief purposes?

A) Shares in a quoted company.

B) Shares in an unquoted company.

C) A business consisting wholly or mainly for making or holding investments.

D) Land or buildings, machinery or plant owned by transferor but used for business purposes by either a company of which the transferor has control, or a partnership of which the transferor was a partner.

E) A business or interest in a business eg business of a sole trader or partnership.

A

C) A business consisting wholly or mainly for making or holding investments.

138
Q

A company wishes to issue preference shares to a new shareholder. The preference shares carry a right to participate in profit and capital on winding up. The company currently has only ordinary shares in issue and articles in the form of unamended model articles.

Which of the following statements sets out the shareholder resolutions which are required?

A) A special resolution to disapply pre-emption rights and a special resolution to amend the articles.

B) An ordinary resolution to give directors authority to allot the shares, a special resolution to disapply pre-emption rights, an ordinary resolution to amend the articles.

C) A special resolution to give directors authority to allot the shares, a special resolution to disapply pre-emption rights, a special resolution to amend the articles.

D) An ordinary resolution to give directors authority to allot the shares, a special resolution to disapply pre-emption rights and a special resolution to amend the articles.

E) An ordinary resolution to give directors authority to allot the shares and a special resolution to amend the articles.

A

D) An ordinary resolution to give directors authority to allot the shares, a special resolution to disapply pre-emption rights and a special resolution to amend the articles

Correct. The company has no cap as it has unamended Model Articles. An OR is required to give directors authority to allot as there are more than one class of shares in issue (s 550 does not apply). The preference shares are equity securities (s 560) as they carry a right to participate so pre-emption rights must be disapplied by SR (s 570(1). As the Company has only ordinary shares in issue the articles must be amended by SR to create the rights to the preference shares.

139
Q

A new investor is keen to buy shares in a public limited company (the “Target”) but does not have the necessary funds to purchase the shares without the aid of a bank loan. The bank has requested security for the bank loan from the Target, the Target’s public limited company subsidiary (the “Public Subsidiary”) and the Target’s private limited company subsidiary (the “Private Subsidiary”).

Which would be the best advice to the Target?

A) The security given by the Target, the Public Subsidiary and the Private Subsidiary would be unlawful financial assistance.

B) Only the security given by the Public Subsidiary and the Private Subsidiary would be unlawful financial assistance.

C) Only the security given by the Private Subsidiary would be unlawful financial assistance.

D) Only the security given by the Target will be unlawful financial assistance.

A

A) The security given by the Target, the Public Subsidiary and the Private Subsidiary would be unlawful financial assistance

Correct. By virtue of s 678(1) CA 06 where a person is acquiring or proposing to acquire shares in a public limited company it is not lawful for that company (ie the Target) or a company that is a subsidiary of that company (ie the Public Subsidiary and the Private Subsidiary) to give financial assistance either directly or indirectly for the purpose of the acquisition.

140
Q

A company (the ‘Buyer’) is looking to purchase the entire issued share capital of a private limited company (the ‘Target’) for £1,500,000. The Buyer has an existing wholly owned subsidiary company which is a PLC. The Buyer is proposing to fund the acquisition partly with a loan from the bank of £750,000. In return for the loan, the bank requires security over the assets of the Target, the Buyer and the Buyer’s subsidiary PLC.

Which of the following statements is correct in respect of prohibited financial assistance?

A) The proposed security over the assets of all three companies potentially falls within the prohibited financial assistance provisions.

B) The proposed security over the assets of the Buyer’s subsidiary PLC potentially falls within the prohibited financial assistance provisions.

C) The proposed security over the assets of the Target potentially falls within the prohibited financial assistance provisions.

D) The proposed security over the assets of the Buyer and the Buyer’s PLC subsidiary potentially falls within the prohibited financial assistance provisions.

E) None of the proposed security would fall within the prohibited financial assistance provisions.

A

E) None of the proposed security would fall within the prohibited financial assistance provisions.

Correct. As the target company is a private limited company, ONLY PLC subsidiaries of the TARGET are caught by the prohibited financial assistance provisions.

141
Q

A private limited company with unamended model articles has 100 ordinary £1 shares as its issued share capital. The company would like to issue an additional 100 ordinary £1 shares to a new shareholder as quickly as possible, so ideally without needing to obtain shareholder approval.

Would it be possible to issue the shares without passing any shareholder resolutions?

A) No, since an ordinary resolution is required to disapply pre-emption rights in relation to the new shares.

B) Yes, since the directors will have automatic authority to issue the new shares because the company has only one class of shares in issue and the new shares to be issued are of the same class.

C) No, since an ordinary resolution giving the directors authority to allot the new shares is required together with an ordinary resolution to disapply pre-emption rights.

D) No, since an ordinary resolution giving the directors authority to allot the new shares is required.

E) No, since a special resolution is required to disapply pre-emption rights in relation to the new shares.

A

E) No, since a special resolution is required to disapply pre-emption rights in relation to the new shares.

Correct. As the company has model articles there is no limit on the number of shares that can be issued so no shareholder approval is needed in respect of that, the directors have authority to allot the new shares under s550 CA 06 as the company only has one type of shares in issue and more of the same are being issued. As the shares are ordinary shares there is no limit on the amount of dividends that the shareholder might receive and no limit on the amount of capital on a solvent winding up of the company so the shares are equity securities by virtue of s560 CA 06. This means to protect the existing shareholders rights of pre-emption can only be disapplied by those existing shareholders passing a special resolution.

142
Q

**A private limited company, which was incorporated in January 2012, currently has an issued share capital of £1000 made up of 1000 £1 ordinary shares. The shares are held equally by four shareholders.

The company now wishes to issue 150 £5 1% preference shares to a fifth shareholder. The preference shares entitle the shareholder to receive only a fixed dividend of £0.01 per share, with no right to share in any surplus profits. Further the shareholder is only entitled to the return of the nominal value of the shares on winding up of the company.

The company has Model Articles with one amendment; this new preference share has already been included in the Articles.

Which of the following statements represents the relevant shareholder resolution(s) required to be passed in order to make the proposed share allotment?**

A) The company can allot the preference shares without obtaining any further authority from the shareholders.

B) The company will need to pass a special resolution to amend the articles of association of the company.

C) The company will need to pass an ordinary resolution to remove or increase the authorised share capital.

D) The company will need to pass a special resolution to disapply pre-emption rights which attach to the shares.

E) The directors of the company will require an ordinary resolution to authorise them to allot the preference shares.

A

E) The directors of the company will require an ordinary resolution to authorise them to allot the preference shares.

Correct. As this is NOT a situation where s 550 CA 2006 applies the company will need further authorisation to allot shares under s 551 CA 2006. The company is a CA 2006 company with no relevant amendments in the articles so there will be no limit on the authorised share capital. The new shares are not equity securities so there is no need to disapply pre-emption rights. The new preference shares are already included in the articles so there is no need to amend the articles

143
Q

A person has taxable income of £45,000. In the same tax year, after the deduction of their annual exemption, they have taxable chargeable gains of £15,000. They have no capital losses for capital gains tax (CGT) purposes. The basic rate tax band for the relevant tax year is £0 - £37,700. The two rates of CGT for the relevant tax year are 10% and 20%.

Based on the above information, what is the person’s CGT liability?

£1,800

£3,000

£1,500

£10,460

£900

A

£3,000

Correct. The 15,000 chargeable gain is taxed at 20%.

144
Q

A person has a taxable income of £200,000. They recently made a taxable chargeable gain of £270,000 on the sale of the shares of an engineering company which they founded 10 years ago. The person held the shares since the incorporation of the company and was a director for the whole of that period, retiring at the date of disposal of the shares. The shares represented a 10% shareholding in the company. The person has not previously made any chargeable gains.

Will the person be able to claim any tax relief on this gain?

A) Business Assets Disposal Relief will apply. The person will not pay any capital gains tax on the gain.

B) No tax reliefs are available. Business Assets Disposal Relief will not apply since the person had only a 10% shareholding.

C) No tax reliefs are available. Business Assets Disposal Relief will not apply since the person has now retired as a director of the company.

D) Business Assets Disposal Relief will apply to reduce the capital gains tax payable from 20% to 10% of the value of the gain.

E) Business Assets Disposal Relief will apply to reduce the income tax payable from 20% to 10% of the value of the gain.

A

D) Business Assets Disposal Relief will apply to reduce the capital gains tax payable from 20% to 10% of the value of the gain.

Correct. This disposal fulfils the criteria for BADR to apply.

145
Q

A person is a partner in a small catering business. The following information relates to their income and other related tax affairs:

Trading profits for tax purposes £73,000

Savings related income £1,800

Gift of a painting from their grandmother £5,000 (market value)

Dividend on shares in a listed company £1,000

Contributions made by the person into a personal pension scheme £5,000

Interest on a loan the person took out to inject further capital into the business £3,500

What is the person’s Net Income?

£70,800

£67,300

£54,800

£75,800

£72,300

A

£67,300

Correct. The Net Income is calculated by calculating the Total Income and deducting available tax reliefs such as interest on qualifying loans and pension contributions.

146
Q

A person owns shares in a private limited company. The shares are not in an ISA. In the current tax year the person receives a dividend of £3,000. The person’s only other source of income in the tax year is their salary. Their taxable income, including the dividend, is £43,000.

The personal allowance for the current tax year is £12,570.

The basic rate tax band of 8.75% for the current tax year applies to dividend income up to and including £37,700 with the higher rate of 33.75% applicable from 37,701 to £125,140. The dividend allowance is £1,000.

How much tax will the person pay on their dividend income?

£0

£800

£675

£1012

£262

A

£675

Correct. You have correctly identified that the taxable income minus the dividend is £40,000, therefore the higher rate applies to the dividend. The first £1,000 of the dividend income is taxed at 0% and the remaining £2,000 will be taxed at the higher rate for dividends of 33.75%.

147
Q

A person disposed of only one chargeable asset in the 2023/24 tax year, which was sold for £100,000. They bought the asset ten years ago for £45,000 with acquisition costs of £5,000. There was no subsequent expenditure before sale. The annual exemption for 2023/24 is £6,000. Sale costs were £2,700. The capital gains tax (‘CGT’) rate for basic rate taxpayers in 2023/24 is 10%, and for higher rate taxpayers it is 20%.

The person’s Taxable Income for 2023/24 was £50,000.

How much capital gains tax will the person have to pay on the disposal of the asset?

£8,800

£4,130

£8,260

£18,585

£9,460

A

£8,260

Correct. The net proceeds of sale will be £97,300 (£100,000 minus £2,700 sale costs). After deducting initial acquisition cost (£45,000) and incidental costs of the acquisition of £5,000, the chargeable gain will be £47,300. The taxable chargeable gain will be £47,300 minus the annual exemption (£6,000) = £41,300. As CGT is taxed after income, and we are told the person is a higher rate taxpayer with a taxable income of £50,000, all of the taxable chargeable gain of £41,300 will be taxed at the higher rate of 20%. The tax liability will therefore be £8,260.

148
Q

For VAT purposes, how are supplies of domestic heating and power rated?

Zero rated

Standard rated at 25%

Reduced rated at 5% VAT

Standard rated at 20%

Exempt

A

Reduced rated at 5% VAT

149
Q

A VAT registered business makes a supply of printer cartridges to a law firm and the contract between them states nothing about VAT.

What is the correct position with regards to VAT payable on the price of the supply of paper?

The price of the supply of the printer cartridges is deemed to be exclusive of VAT.

No VAT will be payable as the parties have not expressly agreed to it.

The price of the supply of printer cartridges is deemed to include the VAT.

The supply of the printer cartridges will be reduced rated for VAT.

The supply of printer cartridges will be exempt from VAT.

A

The price of the supply of printer cartridges is deemed to include the VAT.

150
Q

A small business makes soap using oils purchased from a supplier for £500 plus VAT. They then sell the soap for £5000 plus VAT.

How much VAT will the soap maker send to HMRC?

£0

£1000

£1100

£100

£900

A

£900

151
Q

During its last financial year, ABC Ltd purchased freehold office premises at a cost of £160,000. In order to do this, it took out a loan, on which it paid interest of £10,500 during the year.

The premises were refitted a few years ago, but the décor is fairly traditional so ABC Ltd had the existing fittings ripped out and improvements made to bring the décor into line with ABC’s modern image, at a cost of £14,000.

Whilst the improvements were being made, ABC Ltd rented temporary premises on a short-term lease, paying £35,000 in rent. In the rented premises, there was little space for client entertainment so ABC Ltd took some of the company’s clients out to races, hiring a box and paying for a top-notch caterer to provide canapés and champagne, at a total cost of £8,000.

ABC Ltd had trading receipts of £555,000 in the relevant financial year and other deductible expenditure in the sum of £105,000. It made no disposals of chargeable assets during the year.

What is ABC Ltd’s TTP for the year?

£404,500

£390,500

£76,855

£396,500

£244,500
Correct
Correct. The calculation is as follows:
The company’s TTP is made up of its trading income:
Trading receipts: £555,000
Less deductible expenditure: (£105,000+10,500+35,000= £150,500)
TTP = £404,500 (555,000-150,500)

A

£404,500

152
Q

XYZ Ltd, a manufacturer of scientific equipment, intends to update its production line facilities this year and will shortly pay £345,000 to a supplier of used machine tools.

XYZ Ltd has asked you to advise on how capital allowances can apply in calculating its tax liability in coming years. XYZ Ltd has already used the whole of its annual investment allowance for this year. Its accounting year matches the financial year (1 April - 31 March).

Taking the current financial year as “Year 1”, and assuming that the applicable rates remain the same, which of the following figures is the capital allowance available to XYZ in Year 3?

£41,756

£50,922

£62,100

£190,222

£231,978

A

£41,756

The calculation works out as follows:
Year 1 Writing Down/Capital Allowance (WDA/CA) = (18% x £345,000) = £62,100 Written Down Value (WDV) = (£345,000 - £62,100) = £282,900
Year 2 WDA / CA = (18% x £282,900) = £50,922 WDV = (£282,900 - £50,922) = £231,978
Year 3 WDA / CA = (18% x £231,978) = £41,756 WDV = (£231,978 - £41,756) = £190,222

153
Q

In the financial year a company has trading receipts of £2,212,500 and deductible expenditure of £596,000. It also disposes of a chargeable asset, making a chargeable gain of £610,000. The company has expenditure qualifying for the full annual investment allowance of £1,000,000 during the year and is entitled to additional capital allowances of £12,600.

What is the company’s corporation tax liability for that financial year (rounding down your calculation to the nearest pound)?

£306,625

£556,625

£1,213,900

£553,475

£303,475

A

£303,475

The company’s TTP is made up of its trading income and chargeable gain:
Trading receipts: £2,212,500
Less deductible expenditure: (£596,000)
Less AIA (£1,000,000)
Less CA (£12,600)
Plus Chargeable Gain £610,000
TTP = £1,213,900
@25% = £303,475

154
Q

XYZ Ltd is a close company.

It has made a loan of £28,000 to a participator who is also a director of the company.

Which of the following statements concerning the tax treatment of the loan is correct?

The participator must pay income tax as if they had received a dividend.

There are no tax implications as the loan is below the threshold required.

The company must pay corporation tax at a rate of income tax payable in dividends by a higher rate tax payer.

The participator must pay income tax on the loan at the rate of income tax payable in dividends by a higher rate tax payer.

The company must pay corporation tax at a rate of income tax payable in dividends by a basic rate tax payer.

A

The company must pay corporation tax at a rate of income tax payable in dividends by a higher rate tax payer.

155
Q

ABC Ltd has 7 directors: A, B, C, D, E, F and G.

A and B each own 35% of the shares in the company and the remaining 30% are owned by other individuals (not the other directors).

Which of the following statements concerning whether ABC Ltd is a close company represents the best analysis?

ABC is a close company only because it is under the control of any number of participators who are also directors.

ABC is a close company only because it is under the control of five or fewer participators.

ABC is a close company on both tests.

It is not possible to say whether ABC is a close company as we do not know who the other shareholders are.

ABC is not a close company on either test.

A

ABC is a close company on both tests

The company is under the control of five or few participators (A and B) and under the control of any number of participators who are also directors (A&B).

156
Q

A company has a Total Taxable Profit of £1,340,000 for the tax year ending 5 April 2022.

Which of the following statements best describes how the company must pay its tax liability to HMRC?

A) The company will calculate its tax liability and pay HMRC in two instalments over the course of the next two accounting periods.

B) The company will calculate its tax liability and pay HMRC in four instalments over the course of the next two accounting periods.

C) The company will calculate its tax liability and immediately pay HMRC any tax which is due.

D) The company will calculate its tax liability and pay HMRC within 9 months of the end of the accounting period.

E) The company will calculate its tax liability and pay HMRC within 9 months and one day of the end of the accounting period.

A

E) The company will calculate its tax liability and pay HMRC within 9 months and one day of the end of the accounting period.

157
Q

A woman makes some soy wax from plants grown in her greenhouse and sells it to a candle making company for £2,000 + VAT. The company makes some candles with it and sells all the candles made with the wax to a distributer for £4,000 + VAT. The distributor sells the candles to a retail outlet for £5,500 plus VAT.

How much VAT will be sent to HMRC as a result of the above transactions?

VAT is charged at 20%.

£300

£1,900

£5,500

£1,100

£2,300

A

£1,100

Correct. The amount in each transaction that is paid to HMRC is the difference between the input tax and the output tax.

158
Q

A private interior design company specialises in penthouse apartments and loft conversions. During its last financial year, the company purchased freehold office premises at a cost of £160,000. In order to do this, it took out a loan, on which it paid interest of £10,500 during the year. The company refitted the premises, which were not in keeping with its design criteria, at a cost of £14,000. Whilst the improvements were being made, the company rented temporary premises on a short-term lease, paying £35,000 in rent. The company also incurred business entertainment expenses of £8,000.

The company had trading receipts of £555,000 in the relevant financial year and other deductible expenditure in the sum of £105,000. It made no disposals of chargeable assets during the year.

What is the company’s TTP for the year?

£404,500

£101,125

£244,500

£396,500

£390,500

A

£404,500

Correct. This is calculated as follows:
Trading receipts £555,000
Less deductible expenditure £105,000
Less interest paid on loan £10,500
Less rent £35,000
Total £404,500

159
Q

A private limited company started trading in February two years ago. It has chosen the calendar year as its financial year, so its first accounting period ended on 31 December of the year in which it commenced trading. During this first accounting period, the company made a trading loss of £50,000 in September. The company made no chargeable gains or losses during this first accounting period.

In its second accounting period, which ended on 31 December the following year, the company made a trading profit of £25,000. The company made no capital gain or losses during the second accounting period.

Can the trading profit of £25,000 made in the second accounting period be set off against the trading loss of £50,000 from the first accounting period?

Yes, because no trading or capital losses were incurred in the first accounting period.

No, because the company was not carrying on business for a full 12 month period before the accounting period in which the trading loss was incurred.

No, because a trading loss can only be set off against trading profits from an earlier accounting period.

Yes, because there were no trading profits or chargeable gains in the first accounting period to offset the £50,000 trading loss.

No, because a trading loss can only be set against trading profits or chargeable gains in the same accounting period.

A

Yes, because there were no trading profits or chargeable gains in the first accounting period to offset the £50,000 trading loss.

Correct. The trading loss in the first accounting period has first to be set off against any trading profits or chargeable gains in the same accounting period. However, as there were no trading profits or chargeable gains in the first accounting period, the trading loss could then be offset against future trading profits or chargeable gains in subsequent accounting periods, so can be set off against the £25,000 trading profit in the second accounting year.

160
Q

You have a corporate client which sold some land in September 2022 for £175,000, making a chargeable gain of £25,000. In November 2020, it bought some machinery for £60,000. In May 2023, the client buys new premises for £250,000.

Which statement below represents the best advice to the client regarding the applicability of tax reliefs?

The client can deduct the chargeable gain of £25,000 against either the cost of the machinery or the premises which will help it to reduce the amount of corporation tax it has to pay.

The client can deduct the chargeable gain of £25,000 from the price of the premises only to give a new base cost for the premises of £225,000

The client cannot deduct the chargeable gain of £25,000 against the price of the premises as the premises was not purchased within the qualifying time limit.

The client can deduct the chargeable gain of £25,000 against the price of the premises to give a new base cost for the premises of £225,000 or against the price of the machinery to give a new base cost for the machinery of £35,000.

The client can add the chargeable gain of £25,000 onto the purchase price of the premises to reduce any chargeable gain arising from a future sale of the premises.

A

The client can deduct the chargeable gain of £25,000 from the price of the premises only to give a new base cost for the premises of £225,000

Correct. This represents the operation of roll over relief to defer the payment of a chargeable gain tax liability.